Крок 3 - Медицина 2021 весна 1 день (буклет)

1 / 147
Хворий чоловік 38 років без свідомості. Від родичів відомо, що він хворіє на цукровий діабет і отримує інсулін. Під час огляду встановлено: пульс - 108/хв, слабкого наповнення, AT - 90/60 мм рт. ст. Язик сухий, червоний. Дихання - 20/хв, шумне. Арефлексія. Які обстеження слід провести насамперед? A sick 38-year-old man is unconscious. It is known from his relatives that he has diabetes and receives insulin. During the examination, it was found: pulse - 108/min, weak filling, AT - 90/60 mm Hg. The tongue is dry, red. Breathing - 20/min, noisy. Areflexia. What examinations should be performed first?

Визначення рівня холестерину крові Determination of blood cholesterol level

Визначення рівня ацетону сечі Urine acetone level determination

Визначення рівня цукру сечі Urine sugar determination

Визначення рівня цукру в крові Blood sugar determination

Визначення рівня pH крові Blood pH level determination

2 / 147
До хірурга поліклініки звернувся хворий чоловік 60 років із колото-різаною раною лівої ступні, отриманої п’ять днів тому під час роботи на городі. Рану обробляв розчином йоду. На 5 -й день стан погіршився: підвищилась температура тіла до 38,2°С, з’явився головний біль, підвищена дратівливість, пітливість, біль і сіпання м’язів навколо рани, пізніше з’явились озноб і неможливість відкрити рот. Про яке захворювання необхідно думати? A sick 60-year-old man with a stab wound on his left foot, received five days ago while working in the garden, turned to the polyclinic surgeon. The wound was treated with iodine solution. On the 5th day, the condition worsened: body temperature rose to 38.2°C, headache, increased irritability, sweating, pain and twitching of the muscles around the wound appeared, later chills and the inability to open the mouth appeared. disease must be thought?

Правець Tetanus

Ерізепілоід Erysepiloid

Флегмона Phlegmon

Бешиха Beshikha

Сказ Rabies

3 / 147
У хлопчика віком 4,5 роки після вживання диких ягід з’явилась гіпертермія, шкірний висип, сухість слизових оболонок, мідріаз, гіперемія шкіри, тахікардія. З метою надання невідкладної допомоги необхідно застосувати: A 4.5-year-old boy developed hyperthermia, skin rash, dry mucous membranes, mydriasis, skin hyperemia, tachycardia after eating wild berries. In order to provide emergency aid must be applied:

Атропіну сульфат Atropine sulfate

Унітіол Unithiol

Прозерин Prozerin

Ацетилхолін Acetylcholine

Інфузійну терапію Infusion therapy

4 / 147
У вагітної жінки 29 років установлений діагноз: вагітність 1, 32 тижня. Прееклампсія тяжкого ступеня. Який із наведених препаратів слід призначити для профілактики виникнення судом? A 29-year-old pregnant woman is diagnosed with pregnancy 1, 32 weeks. Severe preeclampsia. Which of the following drugs should be prescribed to prevent seizures?

Клонідину сульфат Clonidine sulfate

Ніфедипін Nifedipine

Бензогексоній Benzohexonium

Сульфат магнію Magnesium sulfate

Діазепам Diazepam

5 / 147
Під час проведення профілактичних заходів на одному із блоків АЕС, співробітник через необережність порушив техніку безпеки, унаслідок чого 35 хвилин перебував у зоні радіоактивного випромінювання. Показання індивідуального дозиметра - 1,0 Гр. Скарги не висуває. Шкірні покриви й видимі слизові оболонки звичайного кольору. Пульс - 84/хв., AT - 138/82 мм рт.ст. Живіт під час пальпації м’який, безболісний. Сформулюйте прогноз життя цього співробітника: During preventive measures at one of the NPP units, an employee negligently violated safety regulations, as a result of which he was in the radioactive radiation zone for 35 minutes. Individual dosimeter reading - 1 ,0 Gy. No complaints. Skin and visible mucous membranes of normal color. Pulse - 84/min., AT - 138/82 mm Hg. Abdomen during palpation is soft, painless. Formulate the life prognosis of this employee:

Сумнівний при відсутності лікування Doubtful in the absence of treatment

Сприятливий при негайному лікуванні Favorable with immediate treatment

Не сприятливий Not favorable

Сумнівний Doubtful

Абсолютно сприятливий Absolutely favorable

6 / 147
Під час проведення реанімації у пацієнта із раптовою зупинкою серця у палаті стаціонару протягом 5 хвилин спостерігається рефрактерна асистолія. Раптом реаніматолог отримав інформацію, що пацієнту перед зупинкою серця випадково було введено внутрішньовенно 5 мл 4% - го калію хлориду замість 0,9% - го хлориду натрію. Який препарат потрібно негайно додати до комплексу реанімаційної заходів? During resuscitation of a patient with sudden cardiac arrest in a hospital ward, refractory asystole is observed for 5 minutes. Suddenly, the intensivist received information that the patient was accidentally injected before the cardiac arrest intravenously 5 ml of 4% potassium chloride instead of 0.9% sodium chloride. What drug should be immediately added to the complex of resuscitation measures?

10 мл 10% - го кальцію хлориду внутрішньовенно 10 ml of 10% calcium chloride intravenously

10 мл аспаркаму внутрішньовенно 10 ml of asparkam intravenously

5 мл 3% - го кордарону внутрішньовенно 5 ml of 3% Cordarone intravenously

1 мл 0,025% - го дигоксину внутрішньовенно 1 ml of 0.025% digoxin intravenously

40 мл 40% - ї глюкози внутрішньовенно 40 ml of 40% glucose intravenously

7 / 147
Чоловіку 53 років з еритематозно-бульозною формою бешихи було призначено внутрішньом’язово пеніцилін. Під час проведення підшкірної проби стан хворого раптово погіршився. З’явилися холодний піт, задишка, недостача повітря та через декілька хвилин втрата свідомості. Об’єктивно встановлено: блідість шкірних покривів, пульс ниткоподібний, AT - 65/45 мм рт.ст., тони серця послаблені. Про яке ускладнення необхідно подумати? A 53-year-old man with an erythematous-bullous form of hysteria was prescribed intramuscular penicillin. During a subcutaneous test, the patient's condition suddenly worsened. Cold sweat, shortness of breath appeared , lack of air and, after a few minutes, loss of consciousness. Objectively established: paleness of the skin, thread-like pulse, AT - 65/45 mm Hg, heart tones are weakened. What complication should be considered?

Гіповолемічний шок Hypovolemic shock

Набряк мозку Brain edema

Інфекційно-токсичний шок Infectious-toxic shock

Анафілактичний шок Anaphylactic shock

Тромбоемболія легеневої артерії Thromboembolism of the pulmonary artery

8 / 147
Чоловік 40 років, отримавши травму під час падіння, звернувся по медичну допомогу зі скаргами на різкий біль у животі над лоном, слабкість, виділення крові з уретри. Загальний стан тяжкий, шкіра та видимі слизові оболонки різко бліді, шкіра вкрита холодним липким потом, пульс - 120/хв., AT - 80/60 мм рт. ст. Встановлено діагноз: травма кісток тазу, гематурія. Яка Ваша тактика? A 40-year-old man, having been injured during a fall, sought medical help with complaints of sharp pain in the abdomen above the pubic area, weakness, bleeding from the urethra. General condition severe, the skin and visible mucous membranes are sharply pale, the skin is covered with cold, sticky sweat, pulse - 120/min, AT - 80/60 mm Hg. The diagnosis was made: trauma to the pelvic bones, hematuria. What are your tactics?

Лапароскопія Laparoscopy

Уретроскопія Uretroscopy

Бужування уретри Urethra bulging

Катетеризація сечового міхура Catheterization of urinary bladder

Очікувальна тактика Waiting tactics

9 / 147
Удома чоловік 40 років отримав травму унаслідок падіння з драбини. Скаржиться на сильний біль у лівій кінцівці. Ps - 88/хв., AT - 120/80 мм рт. ст. На ділянці лівої гомілки припухлість м’яких тканин, деформація у верхній і середній третині, рвано-забита рана завдовжки до 5 см у середній третині гомілки. У рану виступають кісткові уламки. Яку першу допомогу слід надати лікарю ШМД? At home, a 40-year-old man was injured as a result of falling from a ladder. He complains of severe pain in the left limb. Ps - 88/min., AT - 120/80 mm Hg Art. In the area of ​​the left lower leg, there is swelling of soft tissues, deformation in the upper and middle thirds, a lacerated wound up to 5 cm long in the middle third of the lower leg. Bone fragments protrude into the wound. What first aid should be given to the MDD doctor?

Анестезія місця перелому, асептична пов’язка Anesthesia of fracture site, aseptic dressing

Іммобілізація, негайне транспортування до стаціонару Immobilization, immediate transport to hospital

Накладення гіпсової пов’язки Applying a plaster cast

Первинна хірургічна обробка рани, іммобілізація Primary surgical wound treatment, immobilization

Іммобілізація, знеболення, асептична пов’язка, транспортування до стаціонару Immobilization, anesthesia, aseptic dressing, transportation to hospital

10 / 147
2-річна дівчинка, яка спостерігається з приводу хвороби Аддісона, потребує проведення апендектомії. Стан за основним захворюванням компенсований, артеріальний тиск - 95/60 мм рт. ст., вміст натрію сироватки - 125 ммоль/л, калію - 4,5 ммоль/л. Який із запропонованих заходів дасть можливість зменшити ризик розвитку адреналового кризу у цієї пацієнтки? A 2-year-old girl, who is being observed for Addison's disease, needs an appendectomy. The condition of the main disease is compensated, the blood pressure is 95/60 mm Hg. , serum sodium content - 125 mmol/l, potassium - 4.5 mmol/l. Which of the proposed measures will make it possible to reduce the risk of developing an adrenal crisis in this patient?

Зменшення дози глюкокортикостероїдів Glucocorticosteroid dose reduction

Збільшення дози глюко кортикостероїдів Glucocorticosteroid dose increase

Відміна мінералокортикоїдів Withdrawal of mineralocorticoids

Призначення комбінованої антибактеріальної терапії Prescription of combined antibacterial therapy

Уведення синтетичного аналогу АКТГ Introduction of a synthetic analogue of ACTH

11 / 147
Хворий чоловік доставлений до реанімаційного відділення без свідомості. З анамнезу відомо, що у хворого протягом 2 годин був виражений пекучий біль за грудиною. Шкірні покриви бліді, акроціаноз. Артеріальний тиск - 60/40 мм рт. ст. На ЕКГ спостерігалася фібриляція шлуночків. Яке ускладнення розвинулось у хворого? A sick man was brought to the intensive care unit unconscious. It is known from the anamnesis that the patient had burning pain behind the sternum for 2 hours. The skin is pale, acrocyanosis. Arterial pressure - 60/40 mm Hg. Ventricular fibrillation was observed on the ECG. What complications developed in the patient?

Блискавичний набряк легень Lightning pulmonary edema

Розрив міокарда Myocardial rupture

Аритмогенний шок Arrhythmogenic shock

Анафілактичний шок Anaphylactic shock

Тампонада серця Cardiac tamponade

12 / 147
Новонароджена дитина народилася у стані важкої асфіксії. Мають місце блідість шкіри, брадикардія, позитивний симптом “білої плями”. Який препарат треба використати для нормалізації судинного об’єму крові у пологовій залі? A newborn child was born in a state of severe asphyxia. Pale skin, bradycardia, and a positive symptom of a 'white spot' occur. What drug should be used to normalize the vascular volume of blood in the delivery room?

Розчин Рінгера лактату Ringer lactate solution

Реополіглюкін Rheopoliglyukin

0,45% - й розчин натрію хлориду 0.45% sodium chloride solution

0,9% - й розчин назрію хлориду 0.9% sodium chloride solution

Неогемодез Neohemodez

13 / 147
Лікар швидкої медичної допомоги, оглянувши дитину віком 5 місяців, за дві години після травми, поставив діагноз: термічний опік окропом II - III ступеню нижніх кінцівок, промежини, спини до 20% опіковий шок II ступеню. Яким буде об’єм медичної допомоги? The emergency medical doctor, after examining a 5-month-old child, two hours after the injury, made a diagnosis: thermal burn with boiling water of the II - III degree of the lower limbs, crotch, back up to 20% second-degree burn shock. What will be the amount of medical assistance?

Новокаїнові блокади, асептична пов’язка на уражені ділянки Novocaine blocks, aseptic bandage on affected areas

Інфузійна терапія Infusion therapy

Охолодження опікової поверхні Cooling of the burn surface

Знеболення, протишокова терапія, накладання асептичної пов’язки Anesthesia, anti-shock therapy, applying an aseptic bandage

Накладання асептичної пов’язки Applying an aseptic bandage

14 / 147
Під час лікування гострого ендометриту із використанням антибіотиків у хворої жінки раптово з’явились набряки шкіри в ділянці губ і шиї, напади кашлю, задуха, стридорозне дихання. Свідомість збережена, шкіра бліда, температура - 36,8° С, пульс - 92/хв., AT - 130/80 мм рт. ст. Який вірогідний діагноз? During the treatment of acute endometritis with the use of antibiotics, the patient suddenly developed swelling of the skin in the area of ​​the lips and neck, attacks of coughing, suffocation, stridorous breathing. Consciousness is preserved , skin is pale, temperature - 36.8° C, pulse - 92/min., AT - 130/80 mm Hg. What is the likely diagnosis?

Анафілактичний шок Anaphylactic shock

Напад бронхіальної астми Bronchial asthma attack

Гострий ларингіт Acute laryngitis

Набряк Квінке Quincke edema

Інородне тіло в дихальних шляхах Foreign body in respiratory tract

15 / 147
Хвора хлопець 16 років захворіла гостро: турбують біль у животі, часті мізерні рідкі випорожнення зі слизом і прожилками крові, тенезми. Об’єктивно встановлено: температура тіла - 38°С, болюча та спазмована сигмовидна кишка. У копрограмі - слиз, лейкоцити, еритроцити. Яке дослідження слід провести для уточнення діагнозу? A sick 16-year-old boy became acutely ill: abdominal pain, frequent scanty liquid stools with mucus and streaks of blood, tenesmus are bothering him. Objectively established: body temperature - 38 °С, painful and spasmodic sigmoid colon. Mucus, leukocytes, erythrocytes are present in the co-program. What research should be carried out to clarify the diagnosis?

Визначення гемоглобіну Determination of hemoglobin

Ректороманоскопію Rectoromanoscopy

Рентгеноскопію шлунка Rentgenoscopy of the stomach

Бактеріологічне дослідження випорожнень Bacteriological examination of feces

Визначення гематокриту 5Неправильный ответ.Не должен был быть отмечен. Definition of hematocrit 5 Incorrect answer. It should not have been marked.

16 / 147
У пацієнтки 28 років під час вагітності (III триместр) та захворювання на гострий гепатит Е стався викидень, після чого різко погіршився стан: з’явилися психомоторне збудження, сплутаність свідомості, “хлопаючий” тремор, агресивність, дсзорієнтованість у часі, тахікардія, блювання “кавовою гущею”, значно зменшилися розміри печінки. Як оцінюється стан хворої лікарем? A 28-year-old patient had a miscarriage during pregnancy (III trimester) and acute hepatitis E, after which her condition deteriorated sharply: psychomotor agitation, confusion appeared consciousness, 'clacking' tremor, aggressiveness, disorientation in time, tachycardia, vomiting 'coffee grounds', the size of the liver has significantly decreased. How does the doctor assess the condition of the patient?

Геморагічний шок Hemorrhagic shock

Гострий реактивний психоз Acute reactive psychosis

Гостра печінкова недостатність Acute liver failure

Септичний шок Septic shock

Шлунково-кишкова кровотеча Gastrointestinal bleeding

17 / 147
У дитини віком 3 тижнів розпочалося блювання ”фонтаном” за ЗО хвилин після годування, велика кількість залишкової їжі в шлунку. Блювотні маси кислі. Апетит хороший. Дитина за-непокоєна, жадібне смоктання пальця. Голодний запор. Яким захворюванням обумовлена блювота? A 3-week-old child began to vomit 'fountain' 30 minutes after feeding, a large amount of residual food in the stomach. Vomiting masses are acidic. Appetite is good. The child restless, greedy finger-sucking. Hunger constipation. What disease causes vomiting?

Мснінгоенцефаліт Msningoencephalitis

Вроджений пілоростеноз Congenital pylorostenosis

Адреногенітальний синдром Adrenogenital syndrome

Стеноз стравоходу Esophageal stenosis

Харчова алергія Food allergy

18 / 147
У хворої жінки 48 років під час неврологічного огляду симптомів органічного ураження не виявлено. Скаржиться на пронизуючий приступ болю, який виникає несподівано та триває декілька секунд, у ділянці правої верхньої щелепи під час чищення зубів. Коли виникає біль, починається слизотеча з носу та виділення великої кількості слини. Який діагноз? A 48-year-old female patient did not reveal any symptoms of organic damage during a neurological examination. She complains of a piercing attack of pain that occurs unexpectedly and lasts a few seconds, in the area of ​​the right upper jaws while brushing the teeth. When the pain occurs, the nose begins to run and there is a lot of saliva. What is the diagnosis?

Тригемінальна невралгія II гілки (верхнєщелепного нерву) Trigeminal neuralgia of the II branch (of the maxillary nerve)

Головпий біль напруги Tension headache

Невралгія великого потиличного нерву Neuralgia of the large occipital nerve

Мігренозний криз Migraine crisis

Транзиторна ішемічна атака Transient ischemic attack

19 / 147
Дівчинка 11 років після тривалого стояння на шкільній лінійці поскаржилася на запаморочення, слабкість. Раптово зблідла, тіло вкрилося холодним липким потом, шкіра набула мармурового відтінку, з’явився ціаноз губ, сопорозний стан, часте поверхневе дихання. Під час огляду шкільним лікарем спостерігається тахікардія, тони серця ослаблені, AT - 50/10 мм рт.ст. Яке положення треба надати хворій для невідкладної допомоги? An 11-year-old girl complained of dizziness and weakness after standing on the school line for a long time. She suddenly turned pale, her body became covered in cold sticky sweat, her skin took on a marble hue, cyanosis appeared lips, soporose state, frequent shallow breathing. During the examination by the school doctor, tachycardia is observed, heart sounds are weakened, AT - 50/10 mm Hg. What position should be given to the patient for emergency care?

Сидячи Sitting

Горизонтальне з опущеними нижніми кінцівками Horizontal with lowered lower limbs

З піднятою головою With a raised head

Горизонтальне з припіднятими нижніми кінцівками Horizontal with raised lower limbs

Покласти на бік Put aside

20 / 147
У новонародженої дитини на 5-й день життя спостерігається стійка втрата маси тіла, блювання, діарея. Є підозра на наявність солевтратної форми адреногенітального синдрому. На ЕКГ спостерігається низький зубець Т. Яке електролітне порушення наявне у цьому разі? On the 5th day of life, a newborn baby has persistent weight loss, vomiting, and diarrhea. There is a suspicion of the presence of a soleustrate form of adrenogenital syndrome. A low wave is observed on the ECG T. What electrolyte disturbance is present in this case?

Гіпонатріємія Hyponatremia

Гіпокаліємія Hypokalemia

Гіпомагніємія Hypomagnesium

Гіпохлоремія Hypochloremia

Гіпокальціємія Hypocalcemia

21 / 147
Хворий чоловік 68 років доставлений до лікарні зі скаргами на блювоту та водянистий пронос безліч разів. Загальний стан тяжкий. Притомний. Температура тіла - 36,2оС. Шкіра холодна, акроціаноз, тургор знижений. Голос захриплий. У легенях жорстке дихання ЧД - 28/хв. Тони серця приглушені, аритмічні. ЧСС - 102/хв, AT - 80/30 мм рт.ст. Судоми литкових м’язів. Хворий не виділяє сечі. Яка причина тяжкого стану хворого? A sick 68-year-old man was brought to the hospital with complaints of vomiting and watery diarrhea many times. The general condition is serious. He is unconscious. The body temperature is 36.2oC. The skin is cold, acrocyanosis, reduced turgor. Voice hoarse. Hard breathing in the lungs. BH - 28/min. Heart sounds muffled, arrhythmic. Heart rate - 102/min, AT - 80/30 mmHg. Calf muscle spasms. The patient does not urinate What is the cause of the patient's serious condition?

Кардіогенний шок Cardiogenic shock

Інфекційно-токсичний шок Infectious-toxic shock

Геморагічний інсульт Hemorrhagic stroke

Гіповолемічний шок Hypovolemic shock

Гостра затримка сечі Acute urinary retention

22 / 147
До мобільного госпіталю доставлений військовослужбовець, який під час артилерійського обстрілу отримав множинні закриті переломи кісток нижніх кінцівок (гомілки, стегна) та кісток тазу. Які заходи треба включити до комплексу інтенсивної терапії для запобігання виникнення ранніх ускладнень травматичної хвороби? A military serviceman who suffered multiple closed fractures of the bones of the lower limbs (shins, thighs) and pelvis during artillery shelling was taken to the mobile hospital. What measures should be included in the complex intensive therapy to prevent the occurrence of early complications of a traumatic disease?

Профілактика пролежнів Prevention of bedsores

Профілактика жирової емболії Prevention of fat embolism

Профілактика гнійно-септичних ускладнень Prevention of purulent-septic complications

Профілактика ускладнень з боку дихальної системи Prevention of complications from the respiratory system

Профілактика синдрому дисемінованого внутрішньосудинного згортання крові Prophylaxis of disseminated intravascular coagulation syndrome

23 / 147
Рядовий 19 років отримав опіки тулуба і верхніх кінцівок під час вибуху ємкості з пальним. Доставлений до медичної роти бригади. Стан середньої важкості. AT - 100/65 мм рт. ст, пульс - 105/хв. На ділянці верхніх кінцівок і грудної клітки визначаються ділянки гіперемії і набряку міхурів, блідості шкіри з утратою больової і тактильної чутливості. Який обсяг допомоги на цьому етапі? A 19-year-old private received burns to his torso and upper limbs during the explosion of a fuel tank. He was taken to the medical company of the brigade. The condition is of medium severity. AT - 100/65 mm Hg . cent, pulse - 105/min. Areas of hyperemia and swelling of bubbles, pallor of the skin with loss of pain and tactile sensitivity are determined in the area of ​​the upper limbs and chest. What is the amount of help at this stage?

Уведення допаміна в/в крапельно Dip administration of dopamine IV

Обробка ран масляним розчином Treatment of wounds with oil solution

Введення дексаметазону Dexamethasone administration

Обробка ран дегазуючою рідиною Treatment of wounds with degassing liquid

Накладення сухої асептичної пов’язки Applying a dry aseptic bandage

24 / 147
Хворий чоловік 47 років захворів гостро - сильна слабкість, висока лихоманка, біль у литкових м’язах. За З доби помітив появу жовтяниці. Печінка + 2 см. Олігурія. Якщо стан хворого буде погіршуватись, який немедикаментозний метод лікування може його врятувати? A sick 47-year-old man became acutely ill - severe weakness, high fever, pain in the calf muscles. Within 3 days he noticed the appearance of jaundice. Liver + 2 cm. Oliguria If the patient's condition worsens, what non-drug method of treatment can save him?

Озонотерапія Ozone therapy

Плазмаферез Plasmapheresis

Лазеротерапія Laser therapy

Гіпербарична оксигенація Hyperbaric oxygenation

Гемодіаліз Hemodialysis

25 / 147
Дитина з масою тіла 3200 г народилась в асфіксії з оцінкою за шкалою Апгар 2 бали. Після тактильної стимуляції дихання та оксигенації через маску самостійне дихання відсутнє. Частота серцевих скорочень - 60/хв із тенденцією до зниження. Проведена інтубація трахеї з переводом на ШВЛ та розпочато непрямий масаж серця. Зазначте місце компресії на грудину новонародженого під час проведення непрямого масажу серця? A child with a body weight of 3200 g was born asphyxiated with an Apgar score of 2. After tactile stimulation of breathing and oxygenation through a mask, there is no spontaneous breathing. Heart rate - 60/min with a tendency to decrease. Tracheal intubation was performed with transfer to mechanical ventilation and indirect heart massage was started. Indicate the place of compression on the sternum of the newborn during indirect heart massage?

На сосковій лінії On the nipple line

Вище соскової лінії Above nipple line

У ділянці верхівки серця At the apex of the heart

У точці з’єднання верхньої та середньої третини грудини At the junction of the upper and middle third of the sternum

У точці з’єднання середньої та нижньої третини грудини, нище соскової лінії At the junction of the middle and lower third of the sternum, below the nipple line

26 / 147
На фармацевтичному підприємстві сталась пожежа. Декілька працівників виявились заблокованими без можливості самостійної евакуації. Засоби індивідуального захисту на випадок пожежі також виявились недосяжними. Коли рятувальники евакуювали їх до безпечного місця, постраждалі мали такі симптоми: запаморочення, порушення зору, нудота, часте дихання, задишка, вишневий колір обличчя. Який вид гіпоксії розвивається за таких умов? A fire broke out at a pharmaceutical plant. Several workers were trapped without the possibility of self-evacuation. Personal protective equipment in the event of a fire was also out of reach. When rescuers evacuated them to a safe place, the victims had the following symptoms: dizziness, impaired vision, nausea, frequent breathing, shortness of breath, cherry complexion. What type of hypoxia develops under such conditions?

Токсична Toxic

Тканинна Fabric

Гемічна Chemical

Гіпоксична Hypoxic

Циркуляторна Circulator

27 / 147
Пацієнтка 28 років протягом 2 тижнів лежить у ліжку, голова піднята над подушкою, покриви обличчя сальні. Тонус м’язів підвищений, на запитання не відповідає, інструкції не виконує. З боку внутрішніх органів - без патології. Артеріальний тиск - 110/70 мм рт.ст., пульс - 62/хв, температура гіла - 36,7°С. Який Ваш попередній діагноз? A 28-year-old female patient has been lying in bed for 2 weeks, her head is raised above the pillow, her face is oily. Muscle tone is increased, she does not answer questions, does not follow instructions From the side of internal organs - without pathology. Blood pressure - 110/70 mm Hg, pulse - 62/min, body temperature - 36.7°C. What is your previous diagnosis?

Апатичний ступор Apathetic stupor

Епілептичний ступор Epileptic stupor

Психогенний ступор Psychogenic stupor

Депресивний ступор Depressive stupor

Кататонічний ступор Catatonic stupor

28 / 147
Хворий чоловік 28 років доставлений до відділення реанімації та інтенсивної терапії без свідомості. З анамнезу відомо: протягом 7 днів хворіє гострою респіраторною вірусною інфекцією, не лікувався. Об’єктивно встановлено: кома І, гіпертермія, ЧД - 48/хв., ЧСС - 130/хв., гіпертензія, РаО2 - 48 мм рт.ст., раСО2 - 68 мм рт.ст. Яке призначення можна назвати найпершим? A 28-year-old sick man was brought to the intensive care and intensive care unit unconscious. It is known from the anamnesis: he has been suffering from an acute respiratory viral infection for 7 days, he was not treated. Objectively established: coma I, hyperthermia, blood pressure - 48/min., heart rate - 130/min., hypertension, PaO2 - 48 mm Hg, PaCO2 - 68 mm Hg. What appointment can be called the first?

Комп’ютерна томографія органів грудної клітки Computer tomography of chest organs

Вимірювання центрального венозного тиску Measurement of central venous pressure

Бактеріологічне дослідження крові Bacterial blood test

Проба тканин на гідрофільність Test of fabrics for hydrophilicity

Переведення на штучну вентиляцію легень Transfer to artificial lung ventilation

29 / 147
Жінка 58 років скаржиться на появу пекучого болю за грудиною. Біль турбує близько 1 доби, помірний за інтенсивністю, має приступоподібний характер, тривалість больового приступу - 20 - 25 хв. На ЕКГ - інверсія зубця Т 0,1 мВ у відведеннях II, III, avF Запідозрений розвиток заднього інфаркту міокарда без елевації сегмента ST. Який із лабораторних показників слід визначити для верифікації діагнозу? A 58-year-old woman complains of a burning pain behind the sternum. The pain bothers her for about 1 day, is moderate in intensity, has an attack-like character, the duration of the pain attack is 20-25 minutes On the ECG - inversion of the T wave of 0.1 mV in leads II, III, avF Suspected development of posterior myocardial infarction without ST segment elevation. Which of the laboratory parameters should be determined to verify the diagnosis?

Лактатдегідрогенази Lactate dehydrogenase

Креатинфосфокіназу Creatine phosphokinase

MB-фракцію креатинфосфокінази MB fraction of creatine phosphokinase

Трансамінази Transaminases

Тропонін (І або Т) Troponin (I or T)

30 / 147
Хворий на цукровий діабет госпіталізований із приводу швидкого погіршення стану: загальмованість, слабкість, сонливість, поліурія, полідипсія, нудота, блювання. Відчувався запах ацетону у видихуваному повітрі. В аналізі сечі - цукор та ацетонові тіла. Яке ускладнення цукрового діабету найбільш вірогідне? A patient with diabetes was hospitalized due to rapid deterioration of his condition: retardation, weakness, drowsiness, polyuria, polydipsia, nausea, vomiting. The smell of acetone was felt in the exhaled air. In urine analysis - sugar and acetone bodies. What is the most likely complication of diabetes?

Отруєння Poisoning

Надниркова кома Adrenal coma

Кетоацидотична кома Ketoacidotic coma

Печінкова кома Hepatic coma

Гіпоглікемічна кома Hypoglycemic coma

31 / 147
У хворого чоловіка 32 років, який доставлений до лікарні з лептоспірозом, розвинувся інфекційно-токсичний шок. В об’єктивних даних є переважання ціанозу шкіряних покривів і слизових, підвищений центральний венозний тиск, знижений AT. Що слід підключити до інтенсивної терапії? A sick 32-year-old man, who was taken to the hospital with leptospirosis, developed infectious-toxic shock. In objective data, there is a predominance of cyanosis of the skin and mucous membranes, increased central venous pressure, reduced by AT. What should be added to intensive care?

Діуретики Diuretics

Коптрікал Coptrical

Гепарин Heparin

Вазодилятатори Vasodilators

Симпатоміметики Sympathomimetics

32 / 147
Під час обстеження дитини отримано аналіз сечі, забарвлений кров’ю. Під час лабораторного дослідження виявлено незмінені еритроцити. Яке, на вашу думку, найбільш імовірне джерело гематурії? During the examination of the child, a blood-stained urinalysis was obtained. Laboratory examination revealed unchanged red blood cells. What do you think is the most likely source of the hematuria?

Клубочки Cupcakes

Петля Генле Loop of Henle

Канальці Channels

Нефрон Nephron

Нижні сечовивідні шляхи Lower urinary tract

33 / 147
Військовий близько 3 годин тому отримав уламкове поранення лівої половини грудної клітки. Стан тяжкий, ейфорія, шкіра бліда. Тони серця глухі, пульс - 130/хв., слабкий, артеріальний тиск - 80/70 мм рт. ст. Рана грудної клітки - 2 см, розташована ліворуч по парастернальній лінії на рівні III міжреберного проміжку. Дихання над лівою легенею послаблене, у нижніх відділах вкорочений перкуторний звук. На Rg- грамі ОГК спостерігається розширення тіні серця. Яка тактика лікування пораненого? About 3 hours ago, the soldier received a shrapnel wound in the left half of the chest. The condition is serious, euphoric, the skin is pale. The heart sounds are dull, the pulse is 130/min., weak , blood pressure - 80/70 mm Hg. Wound of the chest - 2 cm, located on the left along the parasternal line at the level of the III intercostal space. Breathing over the left lung is weakened, in the lower parts there is a shortened percussion sound. On the Rg-gram of the OGK, an expansion is observed shadows of the heart. What are the tactics of treating a wounded person?

ПХО рани, накладання швів, дренування плевральної порожнини PHO wound, suturing, pleural cavity drainage

Накладання оклюзійної пов’язки, дренування плевральної порожнини Applying an occlusive bandage, draining the pleural cavity

Ушивання рани, комплекс реанімаційних заходів Wound suturing, set of resuscitation measures

Реанімаційні заходи, після стабілізації стану - торакотомія Resuscitation measures, after stabilization of the condition - thoracotomy

Невідкладна торакотомія на фоні реанімаційних заходів Urgent thoracotomy on the background of resuscitation measures

34 / 147
Хвора жінка 34 років підвернула на льоду праву стопу. При обстеженні виявлено набряк та болючість під час пальпації в ділянці правого гомілково- ступневого суглобу, крепітація. Лікар швидкої допомоги вирішив провести транспортну іммобілізацію шиною Крамера в середньо фізіологічному положенні нижньої кінцівки. Яке це положення? A 34-year-old sick woman sprained her right foot on ice. During the examination, swelling and tenderness during palpation in the area of ​​the right ankle-foot joint, crepitation. The emergency doctor decided carry out transport immobilization with a Kramer splint in the average physiological position of the lower limb. What is this position?

Відводять стегно на 10°, ногу згинають у колінному суглобі на 10 - 15°, стопу - на 90° The hip is adducted by 10°, the leg is bent at the knee joint by 10 - 15°, the foot - by 90°

Відводять стегно на ЗО°, ногу згинають у колінному суглобі на 10 - 25°, стопу - на 80° The thigh is moved to 3°, the leg is bent at the knee joint by 10 - 25°, the foot - by 80°

Відводять стегно на 20°, ногу згинають у колінному суглобі на 10 - 25°, стопу - на 80° The thigh is adducted by 20°, the leg is bent at the knee joint by 10 - 25°, the foot - by 80°

Відводять стегно на 40°, ногу згинають у колінному суглобі на 10 - 15°, стопу - на 90° The thigh is adducted by 40°, the leg is bent at the knee joint by 10 - 15°, the foot - by 90°

Відводять стегно на 50°, ногу згинають у колінному суглобі на 10 - 25°, стопу - на 80° The thigh is adducted by 50°, the leg is bent at the knee joint by 10 - 25°, the foot - by 80°

35 / 147
Хворий чоловік мешканець Республіки Конго. Захворів гостро: підвищилась температура до 39,0°С, озноб, міалгії, артралгії. За три дні з’явився висип на бокових поверхнях тулуба, кро-вотеча з носа, мелена. Попередній діагноз - геморагічна лихоманка. Найбільш раціональна терапія хворого: A sick man, a resident of the Republic of Congo. He became acutely ill: the temperature rose to 39.0°C, chills, myalgia, arthralgia. In three days, a rash appeared on the sides body surfaces, nosebleeds, melena. The preliminary diagnosis is hemorrhagic fever. The most rational therapy of the patient:

Антибіотики + специфічна вакцина Antibiotics + specific vaccine

Антигеморагічна терапія + симптоматична терапія Antihemorrhagic therapy + symptomatic therapy

Антибіотики + спазмолітики Antibiotics + antispasmodics

Антибіотики + жарознижувальні Antibiotics + antipyretics

Сульфаніламіди + специфічна вакцина Sulfonamides + specific vaccine

36 / 147
Студент медичного університету прибув із країни, неблагополучної щодо захворювання на холеру. Виїхав із цієї країни 2 тижні тому. Почувається добре, скарг немає. Які заходи щодо студента слід провести? A medical student arrived from a cholera-prone country. He left the country 2 weeks ago. He feels well, no complaints. What measures should be taken regarding the student ?

Спостереження 5 днів за містом проживання Observation of 5 days by city of residence

Протиепідемічні заходи не проводяться Anti-epidemic measures are not carried out

Провести бактеріологічне обстеження Conduct bacteriological examination

Провести серологічне обстеження Conduct a serological examination

Госпіталізувати в інфекційне відділення Hospitalize in the infectious department

37 / 147
До відділення інтенсивної терапії доставлений хлопчик 4 років у непритомному стані. Під час огляду встановлено: порушення свідомості за шкалою ком Глазго - 7 - 8 балів. Температура тіла в нормі. НЬ - 90 г/л, лейкоцити - 8,0- 109/л, АЛТ - 314 ОД, ACT - 155 ОД, білірубін загальний - 56 ммоль/л, натрій плазми крові - 154 ммоль/л, креатинін - 186 мкмоль/л, глюкоза крові - 2,7 ммоль/л. З анамнезу відомо, що протягом останніх 3 діб дитина хворіла на ГРВІ. Мама самостійно давала дитині назоферон, аспірин, діазолін. Яке ускладнення виникло у хлопчика? A 4-year-old boy was brought to the intensive care unit in an unconscious state. During the examination, it was found: impaired consciousness according to the Glasgow coma scale - 7 - 8 points. Body temperature is normal . l, blood glucose - 2.7 mmol/l. It is known from the anamnesis that the child had ARVI during the last 3 days. The mother gave the child nasoferon, aspirin, diazolin on her own. What complication did the boy have?

Гіперосмолярна діабетична кома Hyperosmolar diabetic coma

Гіпоглікемічна кома Hypoglycemic coma

Вірусний гепатит Viral hepatitis

Синдром Рея Ray syndrome

Вірусний енцефаліт Viral encephalitis

38 / 147
До породіллі з доношеною вагітністю і раннім виливом навколоплідних вод викликано лікаря швидкої допомоги. Жінка скаржиться на ядуху, біль за грудиною, тремтіння. Пульс - 120/хв., AT - 70/40 мм рт.ст., температура - 38,4°С. У легенях ослаблений подих із наявністю вологих хрипів. Який діагноз? An emergency doctor was called to a woman in labor with a full-term pregnancy and early amniotic fluid leakage. The woman complains of nausea, pain behind the sternum, trembling. Pulse - 120/min., AT - 70/40 mm Hg, temperature - 38.4°C. Breathing is weakened in the lungs with the presence of moist rales. What is the diagnosis?

Емболія навколоплідними водами Amniotic fluid embolism

Інфаркт міокарда Myocardial infarction

Розрив матки Rupture of uterus

Передчасне відшарування нормально розташованої плаценти Premature detachment of a normally located placenta

Гіповолемічнии шок Hypovolemic shock

39 / 147
У пацієнта на 5-ту добу після операції розвинулась клініка тромбоемболії легеневої артерії, раптова коронарна смерть. Який препарат необхідно було насамперед призначати для профілактики цього ускладнення? On the 5th day after surgery, the patient developed pulmonary embolism, sudden coronary death. What drug should be prescribed first of all to prevent this complication?

Вазотоніки Vasotonics

Тромболітики Thrombolytics

Антифермснтні засоби Anti-fermenting agents

Дезагреганти Disaggregants

Низькомолекулярні гепарини Low molecular weight heparins

40 / 147
Хворий чоловік 58 років, який лікувався у стаціонарі з приводу нестабільної стенокардії, під час ходьби раптово втратив свідомість, упав. Об’єктивно встановлено: свідомість і пульсація на центральних артеріях відсутні, зіниці вузькі, на світло не реагують, рідкі малоамплітудні рухи грудної клітки до 8 - 10/хв. AT не визначається. На ЕКГ: синусоїдна крива з частими, різними за формою і амплітудою хвилями до 300/хв. Яке ускладнення виникло у хворого? A sick 58-year-old man, who was treated in a hospital for unstable angina, suddenly lost consciousness while walking and fell. Objectively established: consciousness and pulsation in the central arteries are absent, the pupils are narrow, do not react to light, rare low-amplitude movements of the chest up to 8 - 10/min. AT is not determined. On the ECG: a sinusoidal curve with frequent, different in shape and amplitude waves up to 300/min. What complication arose in the patient?

Фібриляція передсердь Atrial fibrillation

Гостре порушення мозкового кровообігу Acute cerebrovascular accident

Асистолія шлуночків Ventricular asystole

Фібриляція шлуночків Ventricular fibrillation

Повна атріо-вентрикулярна блокада Complete atrio-ventricular block

41 / 147
8-річну жінку з терміном вагітності 23 - 24 тижні було доставлено до відділення інтенсивної терапії пологового будинку. Пацієнтка в стані геморагічного шоку через маткову кровотечу, зумовлену передчасним відшаруванням плаценти. Анамнез зібрати не вдається через важкість стану. Зі слів родичів вагітність перша, до цього моменту була без особливостей. Однак вагітна страждає на артеріальну гіпертензію 11 ст. Об’єктивно встановлено: свідомість відсутня, пульс ниткоподібний, артеріальний тиск виміряти не вдається. За 2 хвилини після прибуття на кардіомоніторі зафіксовано фібриляцію шлуночків. Негайно розпочато компресію грудної клітки. Подальші невідкладні дії лікаря: An 8-year-old woman with a gestation period of 23-24 weeks was brought to the intensive care unit of the maternity hospital. The patient is in a state of hemorrhagic shock due to uterine bleeding caused by premature detachment of the placenta . It is not possible to collect an anamnesis due to the severity of the condition. According to the relatives, this is the first pregnancy, until that moment it was without any special features. However, the pregnant woman suffers from arterial hypertension of the 11th century. It was objectively established: there is no consciousness, the pulse is threadlike, blood pressure cannot be measured. For 2 Minutes after arrival, ventricular fibrillation was recorded on the cardiac monitor. Chest compressions were immediately started. Further immediate actions of the doctor:

Проведення електричної дефібриляції Performing electrical defibrillation

Переливання рсзус-сумісної крові Rszus-compatible blood transfusion

Введення окситоцину, електрична дефібриляція Oxytocin administration, electrical defibrillation

Проведення медикаментозної кардіоверсії Performance of medical cardioversion

Введення адреналіну, дефібриляція протипоказана Injection of epinephrine, defibrillation is contraindicated

42 / 147
До медичної роти 14 ОМБР, яка розгорнута в районі населеного пункту Широкино з І механізованого батальйону та добровольчого батальйону “Айдар” надійшло 16 поранених вогнепальною зброєю та 8 хворих. Який вид медичної допомоги буде наданий пораненим та хворим у функціональних підрозділах медичної роти бригади? 16 wounded by firearms and 8 sick people came to the medical company 14 of the OMBR, which was deployed in the area of ​​the Shirokino settlement, from the 1st mechanized battalion and the volunteer battalion 'Aidar'. What type of medical care will be provided to the wounded and sick in the functional divisions of the medical company of the brigade?

Перша медична та перша лікарська медична допомога First medical and first medical care

Усі види медичної допомоги All types of medical care

Долікарська допомога та невідкладні заходи кваліфікованої медичної допомоги Medical care and emergency measures of qualified medical care

Перша медична та перша лікарська допомога First medical and first medical aid

Перша лікарська допомога та невідкладні заходи кваліфікованої медичної допомоги First medical aid and emergency measures of qualified medical aid

43 / 147
В одного з учасників ліквідації аварії потяга, який перевозив жовтий фосфор, з’явилися болі в шлунку, нудота, блювота. Блювотні маси мають запах часнику. Яку невідкладну допомогу слід надати у разі симптомів отруєння фосфором? One of the participants in the liquidation of the accident of the train carrying yellow phosphorus developed stomach pains, nausea, vomiting. Vomiting masses smell of garlic. What emergency help should be given in case of symptoms of phosphorus poisoning?

Дати хворому випити молоко Give the patient to drink milk

Призначити сечогінні засоби Prescribe diuretics

Призначити знеболюючі ліки Prescribe painkillers

Промити шлунок 0,3% - м розчином мідного купоросу та ввести антагоніст фосфору хлористий кальцій (10% - 10,0) Wash the stomach with a 0.3% solution of copper sulfate and enter the phosphorus antagonist calcium chloride (10% - 10.0)

Призначити проти блювотні ліки Prescribe anti-emetic medication

44 / 147
У хворого чоловіка з тупою травмою живота та переломами ребер з 4 по 10 виникла зупинка кровообігу. Які заходи серцево-легеневої реанімації необхідно запровадити? A sick man with blunt abdominal trauma and rib fractures from 4 to 10 had circulatory arrest. What measures of cardiopulmonary resuscitation should be implemented?

Масаж серця з положенням хворого на животі Heart massage with the patient lying on his stomach

Масаж серця не проводити Do not perform heart massage

Відкритий масаж серця та ІПВЛ Open heart massage and IPV

Вид масажу немає значення Type of massage does not matter

Закритий масаж серця Closed heart massage

45 / 147
У потерпілого чоловіка діагностовано клінічні ознаки напруженого пневмотораксу. Екстрена допомога під час цієї патології - пункція плевральної порожнини у: The injured man was diagnosed with clinical signs of tension pneumothorax. Emergency care during this pathology - puncture of the pleural cavity in:

VІІІ-IX міжребер’ї по задній аксилярній лінії на боці ураження VIII-IX intercostals along the posterior axillary line on the affected side

II міжребер’ї по середньоключичній лінії на боці ураження II intercostal space along the midclavicular line on the affected side

V-VI міжребер’ї по середній аксилярній лінії на боці ураження V-VI intercostal along the middle axillary line on the affected side

VIII-IX міжребер’ї по задній аксилярній лінії на здоровому боці VIII-IX intercostal space along the posterior axillary line on the healthy side

II міжребер’ї по середньоключичній лінії на здоровому боці II intercostal space along the midclavicular line on the healthy side

46 / 147
Хворого чоловіка 33 років доставлено до хірургічного відділення з клінічною картиною геморагічного шоку. Джерелом кровотечі стала виразка 12 палої кишки. На момент огляду ФГДС - кровотеча не продовжується. Обсяг крововтрати склав 35% ОЦК. Протягом 5 годин хворому перелито близько 1 л еритроцитарної маси (взята 3 доби тому), яка сумісна за групою та резус фактором та після проведення біологічної проби. Після цього у хворого виникли тремор, судоми, прискорення пульсу, аритмія, знизився артеріальний тиск. Яке ускладнення виникло у хворого? A 33-year-old male patient was brought to the surgical department with a clinical picture of hemorrhagic shock. The source of the bleeding was a duodenal ulcer. At the time of the FGDS examination, the bleeding did not continue. The amount of blood loss amounted to 35% BCC. Within 5 hours, the patient was transfused with about 1 L of erythrocyte mass (taken 3 days ago), which is compatible by group and Rh factor and after conducting a biological test. After that, the patient developed tremors, convulsions, increased pulse, arrhythmia, decreased blood pressure. What complication did the patient have?

Гемотрансфузійний шок Hemotransfusion shock

Алергічна реакція Allergic reaction

Калієва інтоксикація Potassium intoxication

Тромбоемболія легеневої артерії Thromboembolism of the pulmonary artery

Цитратна інтоксикація Citrate intoxication

47 / 147
Лікаря викликали для допомоги хворому чоловіку, який кілька хвилин тому втратив свідомість і в якого відсутні ознаки життя. Діагностовано раптову зупинку серця. Якою повинна бути по-слідовність перших реанімаційних заходів згідно з рекомендаціями Європейської Ради з рссусцитації 2010 року? A doctor was called to help a sick man who lost consciousness a few minutes ago and who has no signs of life. A sudden cardiac arrest has been diagnosed. What should be the sequence of first resuscitation measures in accordance with the recommendations of the European Council on rssuscitation of 2010?

Установлення причини зупинки кровообігу і її усунення, непрямий масаж серця (С), штучне дихання (В) Establishing the cause of circulatory arrest and its elimination, indirect heart massage (C), artificial respiration (B)

Непрямий масаж серця (С), контроль та відновлення прохідності дихальних шляхів (А), штучне дихання (В) Indirect heart massage (C), control and restoration of airway patency (A), artificial respiration (B)

Введення адреналіну (Д), введення атропіну (Д), непрямий масаж серця (С) Injection of adrenaline (D), introduction of atropine (D), indirect heart massage (C)

Контроль та відновлення прохідності дихальних шляхів (А), введення адреналіну (Д), дефібриляція Control and restoration of airway patency (A), administration of adrenaline (D), defibrillation

Контроль та відновлення прохідності дихальних шляхів (А), штучне дихання (В), введення атропіну (Д) Control and restoration of airway patency (A), artificial respiration (B), administration of atropine (D)

48 / 147
У дитини 5-місячного віку під час ГРВІ та на тлі підйому температури тіла до 39,5°С виник напад клініко-тонічних судом тривалістю 8 хвилин. Вогнищеві неврологічні знаки та інші симптоми неврологічної дисфункції у дитини до нападу та після нього не спостерігались. Після зниження температури судоми не повторювались. У цьому разі найбільш вірогідним є діагноз: A 5-month-old child had an attack of clinical-tonic seizures lasting 8 minutes during ARVI and against the background of a rise in body temperature to 39.5°C. Focal neurological signs and other symptoms of neurological dysfunction were not observed in the child before and after the attack. After the temperature decreased, the seizures did not recur. In this case, the most likely diagnosis is:

Гнійного менінгіту Suppurative meningitis

Вірусного енцефаліту Viral encephalitis

Фебрильних судом Febrile seizures

Епілепсії Epilepsy

Афективно-респіраторного нападу Affective-respiratory attack

49 / 147
До амбулаторії сімейного лікаря доставлено хворого чоловіка, 19 років, який постраждав у ДТП на мотоциклі. У хворого спостерігається струс мозку, множинні переломи ребер та нижніх кінцівок. Загальний стан середньої важкості, хворий у свідомості, стогне від болю, дихання поверхневе, послаблене. ЧД - 30/хв, ЧСС - 102/хв, AT - 105/60 мм. рт.ст. У легенях дихання везикулярне. Який з етапів надання допомоги у цьому разі є найпершим? A sick man, 19 years old, who was injured in a motorcycle accident, was brought to the outpatient clinic of a family doctor. The patient has a concussion, multiple fractures of ribs and lower limbs. General condition of moderate severity, the patient is conscious, moans from pain, breathing is superficial, weakened. BH - 30/min, heart rate - 102/min, AT - 105/60 mm Hg. Vesicular breathing in the lungs. Which of the stages of providing care in in this case is the first?

Штучна вентиляція легень Artificial lung ventilation

Знеболювання Pain relief

Іммобілізація переломів кісток нижніх кінцівок 5Неправильный ответ.Не должен был быть отмечен. Immobilization of bone fractures of the lower limbs 5 Incorrect answer. It should not have been marked.

Трансфузійно-інфузійна терапія Transfusion-infusion therapy

Накладання щільної пов’язки на грудну клітку на вдосі Applying a tight bandage on the chest on the left

50 / 147
У новонародженої дитини, якій попередньо проводилась інфузійна терапія, виявлено гіпонатріємію. Приріст маси за попередню добу складає 130 г, спостерігаються набряки. Як досягти нормального рівня натрію у крові? Hyponatremia was detected in a newborn child who had previously undergone infusion therapy. The weight gain over the previous day was 130 g, edema was observed. How to achieve a normal level of sodium in the blood?

Введенням 3 г солі на добу per os Introduction of 3 g of salt per day per os

Довенним введенням 0,9% - го розчину натрію хлориду By intravenous injection of 0.9% sodium chloride solution

Довенним введенням 4,2% - го розчину натрію гідрокарбонату By intravenous administration of 4.2% sodium bicarbonate solution

Довенним введенням 5% - го розчину натрію хлориду By intravenous administration of 5% sodium chloride solution

Обмеженням рідини Liquid restriction

51 / 147
До медичного пункту доставлений уражений чоловік, який перебував протягом 3 годин у районі ядерного вибуху. Скаржиться на загальну слабкість, головний біль, спрагу, сухість і гіркоту в роті, нудоту, повторну часту блювоту. Хворий млявий, шкіра обли’ччя і шиї гіперемована. За даними індивідуального дозиметра отримав 4,5 Гр. Пульс лабільний, 90 - 96/хв, AT - 80/40 мм рт.ст. Установіть діагноз: An injured man who was in the area of ​​a nuclear explosion for 3 hours was brought to the medical center. He complains of general weakness, headache, thirst, dryness and bitterness in the mouth, nausea, repeated frequent vomiting. The patient is lethargic, the skin of the face and neck is hyperemic. According to the data of the individual dosimeter, he received 4.5 Gy. The pulse is labile, 90 - 96/min, AT - 80/40 mm Hg. Set the diagnosis:

Гостре отруєння Acute poisoning

ГПХ середнього ступеня (2 ст.), період розпалу GPC of medium degree (2nd century), peak period

Комбіноване ураження Combined lesion

ГПХ тяжкого ступеня (3 ст.), первинна променева реакція GPC of severe degree (3rd grade), primary radiation reaction

ГПХ легкого ступеня (1 ст.), період розпалу GPC of mild degree (1 st.), period of fever

52 / 147
0 годин тому у хворого чоловіка з’явився біль у пахвовій западині, підвищилась температура до 39°С. Об’єктивно виявлено: язик сухий, обкладений білим нальотом, хитка хода. У пахвовій западині виявляється болісний, погано контурований лімфовузол, шкіра над ним гіперемована. Які лабораторні методи треба використати для встановлення діагнозу? 0 hours ago, the sick man had pain in the armpit, the temperature rose to 39°C. Objectively it was found: the tongue is dry, coated with a white coating, shaky gait. A painful, poorly contoured lymph node is found in the axilla, the skin over it is hyperemic. What laboratory methods should be used to establish the diagnosis?

Вірусологічні та біохімічні Virological and biochemical

Бактеріологічні Bacteriological

Радіонуклідні Radionuclides

Паразитологічні Parasitological

Імунологічні Immunological

53 / 147
Після проведення медичного сортування у медичній роті була визначена група поранених, які є нетранспортабельними та по гребують тимчасової госпіталізації. До якого з функціональних відділень медичної роти будуть направлені ці військовослужбовці? After conducting medical triage in the medical company, a group of wounded was identified who are non-transportable and in need of temporary hospitalization. To which of the functional departments of the medical company will these servicemen be sent?

Госпітальне відділення Hospital Department

Сортувально-евакуаційне відділення Sorting and evacuation department

Операційно-перев’язочне відділення Operational and dressing department

Відділення спеціальної обробки Department of Special Processing

Підрозділи забезпечення Providing units

54 / 147
У хворого чоловіка 69 років, який страждає на тромбофлебіт вен нижніх кінцівок, виникла клініка тромбоемболії легеневої артерії. Який із запропонованих діагностичних методів найбільш специфічний під час легеневої емболії? A 69-year-old male patient suffering from thrombophlebitis of the veins of the lower extremities developed a pulmonary embolism clinic. Which of the proposed diagnostic methods is the most specific during pulmonary embolism?

Рентгенограма грудної клітки Chest X-ray

Легенева ангіографія Pulmonary angiography

Аналіз газів артеріальної крові Arterial blood gas analysis

Вентиляційне сканування легень Ventilation lung scan

Псрфузійне сканування легень Lung perfusion scan

55 / 147
Хворий чоловік 70 років скаржиться на гострий біль у правій гомілці та стопі. Захворів гостро, явної причини не знає. Під час огляду встановлено: шкіра гомілки та стопи бліда, холодна на дотик, тактильна та больова чутливість збережені. Пульсація на стегновій артерії правої кінцівки збережена, на підколінній артерії та на судинах стоп не спостерігається. Який ймовірний діагноз? A sick 70-year-old man complains of sharp pain in his right lower leg and foot. He fell ill acutely, he does not know the exact cause. During the examination, it was found that the skin of the lower leg and foot is pale, cold to the touch, tactile and pain sensitivity are preserved. Pulsation on the femoral artery of the right limb is preserved, on the popliteal artery and on the vessels of the feet it is not observed. What is the probable diagnosis?

Гострий тромбофлебіт Acute thrombophlebitis

Глибокий флеботромбоз Deep phlebothrombosis

Тромбоз артерії правої кінцівки Thrombosis of the artery of the right limb

Артрит гомілковостопного суглоба Ankle arthritis

Бешихове запалення Beshikov inflammation

56 / 147
У дитини 6 років, яка з 3-річного віку отримує лікування з приводу епілепсії, на прийомі у дільничного педіатра починається великий судомний напад. Найперші дії лікаря: A 6-year-old child, who has been receiving treatment for epilepsy since the age of 3, has a major seizure during an appointment with the district pediatrician. The doctor's first actions:

Провести оксигенотерапію Carry out oxygen therapy

Провести штучну вентиляцію легень Perform artificial ventilation

Забезпечити прохідність дихальних шляхів Ensure patency of airways

Увести сечогінні Enter diuretics

Увести сибазон внутрішньом’язово Inject sibazon intramuscularly

57 / 147
У дитини 1,5 років із септичним шоком виникла кровотеча з місць ін’єкцій, по шлунковому зонду надходить вміст у вигляді “кавової гущі’.’ Під час визначення показників гемостазу збіль-шено МНВ, АЧТЧ, рівень D-димеру, знижений фібриноген, тромбоцити - ЗО • 109/л. Яку патологію запідозрив лікар-куратор? A 1.5-year-old child with septic shock is bleeding from injection sites, contents in the form of 'coffee grounds' are coming through the stomach probe.' During the determination of hemostasis indicators, LVH, AChT, D-dimer level, decreased fibrinogen, platelets - ZO • 109/l were increased. What pathology did the attending physician suspect?

Тромботична тромбоцитопенічна пурпура Thrombotic thrombocytopenic purpura

Синдром ДВЗ DVZ syndrome

Хвороба Віллебранда Willebrand's disease

Ідіопатична тромбоцитопенічна пурпура Idiopathic thrombocytopenic purpura

Дефіцит вітаміну К Vitamin K deficiency

58 / 147
До окремої медичної роти з гірської місцевості доставлено військовослужбовця. Потерпілий загальмований, байдужий до оточення, мова сплутана. Скаржиться на запаморочення, слаб-кість; є порушення реальної оцінки обставин. Об’єктивно встановлено: температура тіла - 30°С, ректальна - 35,5°С. Шкіра бліда, холодна на дотик, м’язовий тонус знижений. Координація рухів порушена, сухожильні рефлекси пригнічені. Пульс-58/хв, ЧДР-12/хв., AT-90/50 мм рт.ст. На ЕКГ - синусова брадикардія, подовження і деформація комплексу QRS, зміщення інтервалу ST, інверсія зубця Т, подовження інтервалу ОТ Установлено діагноз: загальне охолодження організму. Яка стадія переохолодження у потерпілого? A serviceman was delivered to a separate medical company from a mountainous area. The victim is retarded, indifferent to his surroundings, speech is confused. He complains of dizziness, weakness; there is a violation of the real assessment of the circumstances . Objectively established: body temperature - 30°C, rectal - 35.5°C. The skin is pale, cold to the touch, muscle tone is reduced. Coordination of movements is impaired, tendon reflexes are suppressed. Pulse - 58/min, ChDR- 12/min., AT-90/50 mmHg On the ECG - sinus bradycardia, lengthening and deformation of the QRS complex, ST interval shift, T wave inversion, OT interval prolongation The diagnosis was established: general cooling of the body. What is the stage of hypothermia in the victim ?

II II

ІV IV

III III

V V

I I

59 / 147
Хворого чоловіка 25 років дружина знайшла на підлозі в кімнаті без свідомості. Викликано швидку допомогу. Стан вкрай тяжкий, періодично виникають клонічні судоми кінцівок. На шкірі тулуба і кінцівок темно-багряні плями, на тілі - множинна геморагічна зірчаста висипка різної величини. Тони серця глухі, пульс - 140-160/хв., AT - 40/0 мм рт.ст. Дихання поверхневе, 40/хв. Які невідкладні заходи? The wife found a sick man of 25 years unconscious on the floor in the room. An ambulance was called. The condition is extremely serious, clonic convulsions of the limbs occur periodically. The skin of the trunk and limbs is dark - purple spots, on the body - multiple hemorrhagic star rash of different sizes. Heart sounds are dull, pulse - 140-160/min., AT - 40/0 mm Hg. Breathing is shallow, 40/min. What are the emergency measures?

Введення кордіаміну Introduction of Cordiamine

Введення сібазону Sibazone input

Введення доксицикліну Introduction of doxycycline

Проведення копікотомії Carrying out copyotomy

Введення преднізолону та інфузійна терапія Prednisone administration and infusion therapy

60 / 147
Хворий чоловік 56 років перебуває на лікуванні у травматологічному відділенні з приводу перелому лівого стегна. На другий день перебування раптово з’явились розлади дихання, порушення свідомості, висипка на тулубі, шиї. Під час дослідження крові виявлено збільшення ШОЕ, протромбінового часу, зниження тромбоцитів, гематокритного числа. Яке ускладнення виникло у хворого? A 56-year-old man is being treated in the trauma department for a fracture of the left hip. On the second day of his stay, breathing disorders, unconsciousness, and a rash on the body suddenly appeared , neck. During a blood test, an increase in ESR, prothrombin time, a decrease in platelets, and a hematocrit number were found. What complications did the patient experience?

Порушення мозкового кровообігу Disruption of cerebral circulation

Тромбоемболія Thromboembolism

Пневмонія Pneumonia

Жирова емболія Fat embolism

Анафілактичний шок Anaphylactic shock

61 / 147
Унаслідок падіння з висоти 3 метрів чоловік 25 років отримав травму грудного відділу хребта. Скаржиться на біль, що посилюється під час вдиху. У проекції шостого та сьомого грудних хребців м’які тканини набряклі, болючі. Чутливість на периферії збережена. Який діагноз найбільш імовірний? As a result of a fall from a height of 3 meters, a 25-year-old man suffered an injury to the thoracic spine. He complains of pain that worsens during inhalation. In the projection of the sixth and seventh thoracic vertebrae m 'which tissues are swollen, painful. Peripheral sensitivity is preserved. What is the most likely diagnosis?

Неускладнений компресійний перелом тіл шостого - сьомого грудних хребців Uncomplicated compression fracture of the bodies of the sixth - seventh thoracic vertebrae

Перелом суглобових відростків шостого та сьомого грудних хребців Fracture of the articular processes of the sixth and seventh thoracic vertebrae

Перелом остистих відростків шостого та сьомого грудних хребців Fracture of the spinous processes of the sixth and seventh thoracic vertebrae

Забій грудного відділу хребта Bruise of the thoracic spine

Ускладнений компресійний перелом тіл шостого - сьомого грудних хребців Complicated compression fracture of the bodies of the sixth - seventh thoracic vertebrae

62 / 147
Хворий чоловік 83 років, який перебуває в палаті інтенсивної терапії з приводу загострення ішемічної хвороби серця, раптом втратив свідомість. Зафіксовано клінічну смерть. Яка послідов-ність реанімаційних заходів? An 83-year-old man, who is in the intensive care unit due to an exacerbation of coronary heart disease, suddenly lost consciousness. Clinical death was recorded. What is the sequence of resuscitation measures?'

Заходи реанімації за послідовністю А-В-С та дефібриляція Resuscitation measures according to the sequence of A-B-C and defibrillation

Заходи реанімації за послідовністю А-В-С Resuscitation measures in the sequence A-B-C

Заходи реанімації за послідовністю С-А-В Resuscitation measures according to the sequence C-A-B

Покликати спеціаліста з реанімації та почати штучне дихання Call a resuscitation specialist and start artificial respiration

Заходи реанімації за послідовністю В-А-С Measures of resuscitation according to the sequence of B-A-C

63 / 147
Лікар бригади швидкої допомоги надає невідкладну допомогу дівчинці 10 років, яку збила машина. У зв’язку з неадекватністю самостійного дихання проведена інтубація трахеї. Під час про-ведення пробних вдихів за допомогою мішка Амбу виявлено, що дихальні шуми зліва проводяться слабше. Що треба зробити, щоб виправити ситуацію? The doctor of the ambulance team provides emergency aid to a 10-year-old girl who was hit by a car. Due to the inadequacy of independent breathing, tracheal intubation was performed. During the trial breaths with the help of an Ambu bag revealed that the breath sounds on the left are weaker. What should be done to correct the situation?

Підтягнути інтубаційну трубку вверх Pull the intubation tube up

Просунути інтубаційну трубку вниз Push the intubation tube down

Підвищити тиск у дихальних шляхах Increase airway pressure

Підвищити об’єм дихання Increase breathing volume

Переінтубувати трахею Reintubate trachea

64 / 147
Потерпілого чоловіка було витягнуто з автомобіля після дорожньо-транспортної пригоди (ДТП). Скаржиться на біль у шийному відділі хребта, який посилюється під час рухів головою. Укажіть, який метод транспортної іммобілізації потрібен для такого пошкодження: A male victim was pulled from a car after a traffic accident (traffic accident). He complains of pain in the cervical spine, which is aggravated by head movements. Specify which method of transport immobilization is required for such damage:

Положення хворого на жорстких ношах із валиком під шиєю Position of the patient on a rigid stretcher with a roller under the neck

Положення хворого на спині з валиком під головою Position of the patient on his back with a pillow under his head

Іммобілізація шиною Крамера від потилиці до поперекового відділу хребта Immobilization with a Kramer splint from the back of the head to the lumbar spine

Іммобілізація жорстким головоутримувачем Immobilization with a rigid head restraint

Положення хворого на животі на жорстких ношах Position of a patient on his stomach on a hard stretcher

65 / 147
Під час проведення профілактичних заходів на одному з блоків АЕС один із співробітників через необережність порушив техніку безпеки, унаслідок чого він 35 хвилин перебував у зоні радіоактивного опромінення. Показники індивідуального дозиметра - 1,0 Гр. Скарг не висуває. Шкірні покриви та видимі слизові оболонки зберігають свій звичайний колір. Пульс - 84/хв., артеріальний тиск - 138/82 мм рт. ст. Живіт працівника під час пальпації м’який, неболючий. Яких лікувально- профілактичних заходів слід вжити стосовно цього співробітника? During preventive measures at one of the units of the NPP, one of the employees carelessly violated safety regulations, as a result of which he was in the zone of radioactive exposure for 35 minutes. Indicators of the individual dosimeter - 1.0 Gy. No complaints. The skin and visible mucous membranes retain their normal color. Pulse - 84/min., blood pressure - 138/82 mm Hg. The worker's abdomen is soft, painless during palpation. What medical and preventive measures should be taken in relation to this employee?

Призначити антибактеріальну терапію Assign unitiol

Призначити препарати йоду Prescribe iodine preparations

Профілактичного лікування не потребує Does not require preventive treatment

Призначити унітіол Prescribe antibacterial therapy

Призначити цистамін Prescribe cystamine

66 / 147
Під час огляду пораненого солдата на медичному пункті батальйону встановили наявність ліквореї і кровотечі з вуха та носа. Постраждалий без свідомості. Поставте попередній діагноз? During the examination of the wounded soldier at the battalion medical station, the presence of liquefaction and bleeding from the ear and nose was established. The victim is unconscious. Make a preliminary diagnosis?

Забій головного мозку Brain stroke

Перелом склепіння черепа Fracture of the vault of the skull

Перелом основи черепа Skull base fracture

Струс головного мозку Concussion

Здавлення головного мозку Brain Crush

67 / 147
Хворий чоловік 36 років, на місці ДТП оглянутий лікарем швидкої допомоги. Виявлено травму грудної клітки з клінікою гострої крововтрати і геморагічного шоку з симптомами недостатності дихання. Що слід зробити для запобігання погіршення стану потерпілого, якщо госпіталізація затримується? A sick man, 36 years old, was examined by an ambulance doctor at the scene of the accident. A chest injury was detected with a clinic of acute blood loss and hemorrhagic shock with symptoms of respiratory failure. What should be done for preventing deterioration of the victim's condition if hospitalization is delayed?

Падати напівсидяче положення і вводити анальгетики Fall in a semi-sitting position and administer analgesics

Для запобігання набряку вводять діуретики Diuretics are administered to prevent edema

Уводити серцеві глікозиди і дихальні аналептики Inject cardiac glycosides and respiratory analeptics

У плевральну порожнину ввести дренажну грубку Insert a drainage tube into the pleural cavity

Плевральна пункція у VII міжребер’ї та евакуація крові Pleural puncture in the VII intercostal space and blood evacuation

68 / 147
Боєць 28 років доставлений із зони бойових дій до військового польового пересувного госпіталю після отримання травми грудної клітки внаслідок дії вибухової хвилі. Оглушений, неспокійний. Під час загального огляду виявлено: ціаноз обличчя, поверхневе дихання, тахіпное - 27/хв., пульс - 125/хв. У лівій половині грудної клітки по передньо-аксілярній лінії наявна крепітація та підшкірна емфізема. Ваші дії для надання допомоги: A 28-year-old soldier was transported from the combat zone to a military field mobile hospital after receiving a chest injury as a result of the blast wave. He is stunned, restless. During a general examination, it was found: cyanosis of the face, shallow breathing, tachypnea - 27/min., pulse - 125/min. Crepitation and subcutaneous emphysema are present in the left half of the chest along the front-axillary line. Your actions to provide assistance:

Паравертебральна новокаїнова блокада Paravertebral novocaine blockade

Киснева терапія Oxygen therapy

Інфузійна терапія, наркотичні анальгетики Infusion therapy, narcotic analgesics

Пункція плевральної порожнини Puncture of the pleural cavity

Торакоцентез, дренування лівої плевральної порожнини Thoracentesis, drainage of the left pleural cavity

69 / 147
У пацієнта 18 років під час лікування з приводу бактеріального ендокардиту раптово на фоні емоційного напруження розвинувся генералізований судомний напад. Після нападу в неврологічному статусі визначалися: центральний параліч VII, XII пар черепних нервів, геміанестезія та центральний параліч праворуч. Який найбільш імовірний діагноз? An 18-year-old patient, during treatment for bacterial endocarditis, suddenly developed a generalized convulsive attack against the background of emotional stress. After the attack, the following neurological status was determined: central paralysis VII, XII pair of cranial nerves, hemianaesthesia and central paralysis on the right. What is the most likely diagnosis?

Гонералізований епілептичний напад Generalized epileptic seizure

Кардіоемболічний інсульт Cardioembolic stroke

Гомодинамічний інсульт Homodynamic stroke

Лакунарний інсульт Lacunar stroke

Субарахноїдальний крововилив Subarachnoid hemorrhage

70 / 147
До акушерського відділення доставлена першовагітна 26 років, у терміні вагітності 40 тижнів. Скарги на відсутність рухів плода протягом 2 днів. Води не відходили. Під час обстеження вста-новлено: матка відповідає 40 тижням вагітності, у нормотонусі, положення плода поздовжнє, голівка притиснута до площини в малий таз. Серцебиття плода не вислуховується. Яка тактика лікаря? A 26-year-old primiparous woman, 40 weeks pregnant, was brought to the obstetrics department. Complaints about the lack of fetal movement for 2 days. Water did not flow. During the examination, it was established : the uterus corresponds to 40 weeks of pregnancy, in normotonus, the position of the fetus is longitudinal, the head is pressed against the plane of the small pelvis. The heartbeat of the fetus cannot be heard. What are the doctor's tactics?

Спостереження Observations

Консультація терапевта Therapist consultation

Кардіотокографія Cardiotocography

УЗД Ultrasound

Амніотонія Amniotonia

71 / 147
Жінка 25 років скаржиться на плаксивість, різкі зміни настрою, прискорене серцебиття, що зв’язує з втомою - місяць тому народила близнюків. З ранку відчула “зупинки” в серцевій діяльності. AT - 130/70 мм рт. ст., ЧСС- 115/хв., часті екстрасистоли, ЧДР - 17/хв. Щитовидна залоза збільшена до другого ступеня, безболісна. Лікар діагностував післяпологовий тиреоїдит. Яке обстеження найдоцільніше призначити для визначення функції щитовидної залози? A 25-year-old woman complains of tearfulness, sudden changes in mood, rapid heartbeat, which she associates with fatigue - she gave birth to twins a month ago. From the morning she felt 'stops' in heart activity. AT - 130/70 mm Hg, heart rate - 115/min, frequent extrasystoles, CHD - 17/min. The thyroid gland is enlarged to the second degree, painless. The doctor diagnosed postpartum thyroiditis. What examination is most appropriate to prescribe to determine functions of the thyroid gland?

Рівень у крові ТТГ, ТЗ вільний, Т4 вільний Blood level of TSH, free TK, free T4

Радіоізотопна сцинтиграфія щитовидної залози Radioisotope thyroid scintigraphy

УЗД щитовидної залози Ultrasound of the thyroid gland

Рівень у крові антитіл до ТПО The blood level of TPO antibodies

Рівень у крові антитіл до ТГ The blood level of anti-TG antibodies

72 / 147
Під час святкування дня народження одному з гостей раптово стає погано: зблід, кашляє. Присутні підозрюють, що він вдихнув кістку з риби. Найбільш специфічним проявом наявності стороннього тіла гортані є: During the birthday party, one of the guests suddenly feels sick: he turns pale, coughs. Those present suspect that he inhaled a fish bone. The most specific manifestation of the presence of a foreign body in the larynx is:

Напад інспіраторної ядухи Attack of inspiration

Посиніння обличчя Bluish face

Почервоніння обличчя Flush

Напад експіраторної ядухи Attack of expiratory dyspnea

Розвиток гіпотонії Development of hypotension

73 / 147
У хворого чоловіка 58 років, що переніс Q-інфаркт міокарда, з’явилися напади серцебиття, що супроводжуються задишкою, болем за грудиною, загальною слабкістю. Об’єктивно встановлено: акроціаноз, пульс - 160/хв., ритмічний, артеріальний тиск - 105/65 мм рт. с., тони серця ослаблені. На ЕКГ виявлено: зубці Р відсутні, QRS - деформовані, розширені - 0,14 с, зубці Т направлені в протилежну сторону від QRS. Яке порушення ритму розвинулося в хворого? A 58-year-old male patient who suffered a Q-myocardial infarction developed palpitations accompanied by shortness of breath, chest pain, and general weakness. Objectively established: acrocyanosis, pulse - 160/min., rhythmic, blood pressure - 105/65 mm Hg, heart sounds are weakened. ECG revealed: P waves absent, QRS - deformed, widened - 0.14 s, T waves are directed in the opposite direction from the QRS. What rhythm disturbance has developed in the patient?

Фібриляція шлуночків Ventricular fibrillation

Пароксизмальна надшлуночкова тахікардія Paroxysmal supraventricular tachycardia

Пароксизмальна форма фібриляції передсердь Paroxysmal form of atrial fibrillation

Синусова тахікардія Sinus tachycardia

Пароксизмальна шлуночкова тахікардія Paroxysmal ventricular tachycardia

74 / 147
Хворий чоловік 27 років збитий автомобілем. У нижній третині лівої гомілки визначається деформація, патологічна рухливість кісткових сегментів, рана 5x3 см, з якої непульсуючим струменем виділяється темна кров. Який оптимальний метод зупинки кровотечі? A sick 27-year-old man was hit by a car. In the lower third of the left leg, there is a deformity, pathological mobility of bone segments, a 5x3 cm wound, from which dark blood is emitted in a non-pulsating stream. What the optimal method of stopping bleeding?

Стискальна асептична пов’язка Squeezable aseptic bandage

Пальцеве притискання в пахвинній ділянці Finger pressure in the groin area

Накладення джгута вище рани Applying a tourniquet above the wound

Асептична пов’язка Aseptic dressing

Накладення джгута нижче рани Applying a tourniquet below the wound

75 / 147
У дитини 8 років захворювання мало поступовий початок із підвищення температури до 37,5 - 38°С, нежиттю, сухого “гавкаючого” кашлю, осиплості голосу. На третю добу хвороби стан дитини погіршився, з’явилося шумне дихання з втяжінням міжреберних проміжків. Яке захворювання з числа ГРВІ найбільш імовірно у дитини? In an 8-year-old child, the disease had a gradual onset with an increase in temperature to 37.5 - 38°C, dyspnea, dry 'barking' cough, hoarseness of voice. On the third On the day of the illness, the child's condition worsened, noisy breathing appeared with intercostal spaces retracting. Which ARVI disease is the most likely in the child?

Риновірусна інфекція Rhinovirus infection

Аденовірусна інфекція Adenovirus infection

Грип Flu

РС-інфекція MS infection

Парагрип Paraflu

76 / 147
У хворого хлопця 14 років після укусу комахи розвинулись прояви анафілактичного шоку. На другий день захворювання різко зменшився добовий діурез, у біохімічному аналізі крові відмічається приріст сечовини та креатиніну. З чим це пов’язано? A 14-year-old sick boy developed manifestations of anaphylactic shock after an insect bite. On the second day of the disease, the daily diuresis decreased sharply, and an increase in urea and creatinine was noted in the biochemical blood analysis. With what is it connected with?

З розвитком хронічної ниркової недостатності With the development of chronic renal failure

З розвитком гострої ниркової недостатності With the development of acute renal failure

З розвитком гострого гломерулонефриту With the development of acute glomerulonephritis

З розвитком інтерстиційного нефриту With the development of interstitial jade

З розвитком гострої наднирникової недостатності With the development of acute adrenal insufficiency

77 / 147
Хворий чоловік скаржиться на високу температуру, головний біль, біль у м’язах, попереку. Склери іктеричні. Повернувся з Африки тиждень тому. Яка найбільш вірогідна патологія, що зумовлює таку картину? A sick man complains of a high temperature, headache, pain in the muscles, lower back. The sclera is icteric. He returned from Africa a week ago. What is the most likely pathology that causes such a picture?

Малярія Malaria

Грип Flu

Жовта лихоманка Yellow fever

Лептоспіроз Leptospirosis

Гепатит А Hepatitis A

78 / 147
Хвора жінка 20 років прооперована з приводу дифузного токсичного зобу III ст. Після операції у хворої з’явились серцебиття, пітливість, почуття страху, пронос. Шкіра волога, гаряча на дотик, температура - 40°С. Набряки відсутні. Ps - 160/хв., аритмічний, миготлива аритмія, напружений. AT - 170/60 мм рт.ст. Тони серця звучні. Яке ускладнення розвинулось у хворої? A 20-year-old woman was operated on for a diffuse toxic goiter of the 3rd stage. After the operation, the patient developed palpitations, sweating, a feeling of fear, diarrhea. The skin is moist, hot to the touch, temperature - 40°C. There is no edema. Ps - 160/min., arrhythmic, atrial fibrillation, strained. AT - 170/60 mm Hg. Heart sounds are sonorous. What complication developed in the patient?

Гостра респіраторна вірусна інфекція Acute respiratory viral infection

Тиреотоксичний криз Thyrotoxic crisis

Анафілактичний шок Anaphylactic shock

Гіпертонічний криз Hypertensive crisis

Гіпотиреоїдна кома Hypothyroid coma

79 / 147
На прийом до лікаря прийшла мама з 4-річною дитиною зі скаргами на свербіж шкіри та висипку. Із анамнезу відомо, що напередодні дівчинка вживала шоколад. Об’єктивно встановлено: розповсюджений уртикарний висип на шкірі, елементи блідо-рожевого кольору, оточені зоною еритеми, що зникає під час натискання, локально температура тіла підвищена. З боку внутрішніх органів патології немає. Який патологічний стан виник у дитини? A mother with a 4-year-old child came to see a doctor with complaints of itchy skin and a rash. It is known from the anamnesis that the girl had consumed chocolate the day before. It was objectively established : widespread urticarial rash on the skin, elements of pale pink color, surrounded by a zone of erythema that disappears when pressed, locally the body temperature is elevated. There is no pathology from the side of the internal organs. What pathological condition did the child develop?

Гостра кропив’янка Acute urticaria

Атонічний дерма тит Atonic dermatitis

Синдром Стівенса-Джонсона Stevens-Johnson Syndrome

Токсикодермія Toxicoderma

Апгіоневротичний набряк Apgioneurotic edema

80 / 147
У палаті інтенсивної терапії перебуває новонароджена дитина з респіраторним дистрес-синдромом І типу, III ступеня та інтранатальною асфіксією важкого ступеня; недоношеність ЗО тижнів. Вона отримує штучну вентиляцію легень у примусовому режимі. Відзначається порушення гемодинаміки, середній артеріальний тиск - 60/20 мм рт.ст. Призначте найперший препарат у цій ситуації: There is a newborn baby in the intensive care unit with respiratory distress syndrome type I, degree III and intranatal asphyxia of a severe degree; prematurity of 30 weeks. She receives artificial lung ventilation in in a forced mode. Hemodynamic disturbances are noted, the average arterial pressure is 60/20 mm Hg. Prescribe the first drug in this situation:

25% - й розчин MgSО4 1 мл/кг 25% solution of MgSO4 1 ml/kg

Допамін 10 мкг/кг/хв Dopamine 10 μg/kg/min

10% - й розчин глюкози 2 мл/кг 10% glucose solution 2 ml/kg

Фенобарбітал 20 мг/кг Phenobarbital 20 mg/kg

5% - й розчин глюкози 4 мл/кг 5% glucose solution 4 ml/kg

81 / 147
Доношена вагітність, двійня. 5 хвилин тому народився перший плід у головному передлежанні. Під час піхвового дослідження плодовий міхур другого плода цілий. Передлежать ніжки другого плода. Серцебиття плода приглушене - 160/хв. Які подальші дії? Full-term pregnancy, twins. 5 minutes ago, the first fetus was born in breech presentation. During vaginal examination, the amniotic sac of the second fetus is intact. The legs of the second fetus are presented. The fetal heartbeat is muffled - 160/min. What are the next steps?

Амніотомія. Витягання плода за ніжку Amniotomy. Pulling the fetus by the leg

Кесарів розтин Caesarean section

Ведення пологів за Цов’яновим Management of childbirth according to Tsovyanov

Зовнішній поворот плода на голівку External rotation of the fetus on the head

Лікування гіпоксії плода Treatment of fetal hypoxia

82 / 147
Потерпілий чоловік доставлений на сортувальний майданчик медичної роти за 2,5 години після ядерного вибуху. Скаржиться на сильний головний біль, запаморочення, слабкість, яка на-ростає, спрагу, постійну нудоту та блювоту. Перераховані симптоми з’явилися за 5 - 10 хв. після опромінення та неухильно зростали. Під час огляду чоловік в’ялий, малорухомий, спостері-гається гіперемія шкіри, почервоніння склер, продовжується блювота. Пульс - 110/хв., ритмічний, тони серця приглушені, І тон на верхівці ослаблений, AT - 90/70 мм рт.ст., ЧД - 24/хв. Температура тіла - 38,7“С. Показники індивідуального дозиметру - 5,9 Гр. Який діагноз? The injured man was brought to the sorting area of ​​the medical company 2.5 hours after the nuclear explosion. He complains of a severe headache, dizziness, increasing weakness, thirst , constant nausea and vomiting. The listed symptoms appeared 5-10 minutes after exposure and steadily increased. During the examination, the man is lethargic, immobile, hyperemia of the skin, redness of the sclera, vomiting continues. Pulse - 110/min ., rhythmic, heart sounds are muffled, I tone at the apex is weakened, AT - 90/70 mm Hg, BH - 24/min. Body temperature - 38.7'C. Indicators of the individual dosimeter - 5.9 Gy. What diagnosis?

Гостра променева хвороба II (середнього) ступеня важкості Acute radiation sickness II (medium) degree of severity

Гостра форма променевої хвороби - церебральна Acute form of radiation sickness - cerebral

Гостра променева хвороба ІІІ (важкого) ступеня Acute radiation sickness of III (severe) degree

Гостра променева хвороба IV (вкрай важкого) ступеня Acute radiation sickness of the IV (extremely severe) degree

Гостра променева хвороба І (легкого) ступеня Acute radiation sickness of the I (mild) degree

83 / 147
За 5 хвилин після ін’єкції цефазоліну хворий хлопець втратив свідомість, з’явився набряк усього тіла, поліморфний висип, AT - 30/0 мм рт. ст., ЧСС - 120/хв. Застосування якого препарату у цьому разі є патогенетично обумовленим? 5 minutes after the injection of cefazolin, the sick boy lost consciousness, swelling of the whole body appeared, a polymorphic rash, AT - 30/0 mm Hg. , heart rate - 120/min. The use of which drug in this case is pathogenetically determined?

Хлористий кальцій Calcium chloride

Норадреналін Noradrenaline

Димедрол Diphenhydramine

Преднізолон Prednisone

Адреналін Adrenaline

84 / 147
Дитина народилася вдома. Які дії лікаря швидкої допомоги повинні бути найпершими, якщо він бачить доношеного малюка, який голосно кричить, виконує активні рухи, має задовільний м’язовий тонус та явища акроціанозу, ЧСС - 140/хв., ЧД - 54/хв.? The baby was born at home. What should be the first actions of an emergency doctor if he sees a full-term baby who cries loudly, performs active movements, has satisfactory muscle tone and phenomena of acrocyanosis, heart rate - 140/min., HR - 54/min.?

Перевірити прохідність стравоходу Check the patency of the esophagus

Виконати коптакт “шкіра до шкіри” дитини з матір’ю та укрити їх спільною ковдрою (“тепловий ланцюжок”) Perform a skin-to-skin coptact of the child with the mother and cover them with a common blanket ('thermal chain')

Залишити породіллю та дитину вдома Leave the mother and child at home

Госпіталізувати до відділення патології новонароджених Hospitalize to the neonatal pathology department

Негайно транспортувати до пологового будинку Transport immediately to the maternity hospital

85 / 147
Жінка 74 років хворіє на цукровий діабет другого типу 12 років, приймає глібенкламід. Протягом двох тижнів щоденно без контролю лікаря приймає по 2 таблетки гіпотіазиду. П’ять днів тому, після переохолодження, підвищилась температура тіла до 38°С, з’явився біль у горлі, головний біль, нудота, трьохразове блювання. Об’єктивно встановлено: свідомість сплутана, шкірні покриви сухі, теплі, тургор м’язів знижений, температура тіла - 38°С, пульс - 120/хв, AT- 60/40 мм рт. ст., дихання поверхневе, язик сухий, запах ацетону відсутній. Глюкоза крові - 25,4 ммоль/л, ацетон в сечі - слабкопозитивний, Na1 сироватки - 162 ммоль/л, - 3,0 ммоль/л. Який розчин показаний для введення насамперед? A 74-year-old woman has been suffering from type 2 diabetes for 12 years and has been taking glibenclamide. She has been taking 2 hypothiazide tablets daily for two weeks without a doctor's supervision. Five days ago , after hypothermia, the body temperature rose to 38°C, a sore throat, headache, nausea, vomiting appeared three times. Objectively established: consciousness is confused, skin is dry, warm, muscle turgor is reduced, body temperature - 38°C, pulse - 120/min, AT - 60/40 mm Hg, shallow breathing, dry tongue, no smell of acetone, blood glucose - 25.4 mmol/l, acetone in urine - weakly positive, serum Na1 - 162 mmol/l, - 3.0 mmol/l. Which solution is indicated for introduction first?

5% - й розчин глюкози 5% glucose solution

Поляризуюча суміш Polarizing mixture

Фізіологічний розчин NaCl Saline NaCl

Гіпотонічний розчин NaCl Hypotonic NaCl solution

Розчин Рінгера Ringer's solution

86 / 147
Поблизу селища відбуваються бойові дії. Населення району може попасти під вплив бойових дій. Від чого залежить організація лікувально-евакуаційного забезпечення населення в бонових умовах? There are hostilities taking place near the village. The population of the district may be affected by the hostilities. What does the organization of medical and evacuation support of the population depend on in good conditions?

Від прогнозування місця та часу виникнення осередку медико-санітарних втрат From forecasting the place and time of occurrence of a center of medical and sanitary losses

Від медичної підготовки населення From medical training of the population

Від стратегії та заходів ведення бойових дій From the strategy and measures of military operations

Від кількості захисних споруд From the number of protective structures

Від розміру та структури медико-санітарних втрат населення From the size and structure of medical and sanitary losses of the population

87 / 147
Пацієнт чоловік 56 років протягом 50 годин перебуває на штучній вентиляції легень, що проводиться через ендотрахеальну трубку, у нього сильна лихоманка, артеріальний тиск - 85/40 мм рт. ст., частота серцевих скорочень - 120/хв., PaO2/FiO2 - 260 мм рт. ст., центральний венозний тиск - 12 мм вод.ст., лейкоцитоз (18 Г/л), паличкоядерний зсув (18%). Якій причині сепсису відповідає цей діагноз? A 56-year-old male patient has been on artificial lung ventilation through an endotracheal tube for 50 hours, he has a high fever, blood pressure is 85/40 mm Hg. Art., heart rate - 120/min., PaO2/FiO2 - 260 mmHg, central venous pressure - 12 mmHg, leukocytosis (18 G/L), rod-nuclear shift (18%). What does this diagnosis correspond to the cause of sepsis?

Вентиляційне-асоційована пневмонія Ventilation-associated pneumonia

Синусит Sinusitis

Гострий безкам’яний холецистит Acute stoneless cholecystitis

Інтраабдомінальний сепсис Intra-abdominal sepsis

Інфекція, пов’язана з центральним венозним катетером Central venous catheter-related infection

88 / 147
Хворий чоловік 32 років без постійного місця проживання звернувся до лікаря на 5-й день хвороби зі скаргами на гарячку, сильний біль голови, безсоння. Об’єктивно встановлено: температура тіла - 40°С, пульс - 110/хв. Хворий збуджений, говіркий. Лице гіперемоване. Склерит. На тулубі рясна розеольозно-петехіальпа висипка. Позитивний симптом Говорова-Годельє. Пальпуються збільшені печінка й селезінка. Що потрібно з’ясувати в епіданамнезі? A sick 32-year-old man without a permanent place of residence consulted a doctor on the 5th day of illness with complaints of fever, severe headache, insomnia. Objectively established: body temperature - 40°C, pulse - 110/min. The patient is agitated, talkative. The face is hyperemic. Scleritis. There is a profuse roseolosis-petechialpa rash on the trunk. Positive Govorov-Godelier symptom. An enlarged liver and spleen are palpated. What should be found out in the epidemiologic history ?

Контакт із гризунами Contact with rodents

Споживання недоброякісної їжі Consumption of poor quality food

Наявність парентеральних втручань Presence of parenteral interventions

Наявність вошей Presence of lice

Укус невідомого собаки Bite of unknown dog

89 / 147
У новонародженої дитини, стан якої у попередні дні був задовільним, на 3-й день життя з’явилась кровотеча зі шлунка, домішки свіжої крові в калі. Вітамін К і у першу добу життя не вводився. Про який найімовірніший діагноз слід подумати? A newborn child, whose condition was satisfactory in the previous days, on the 3rd day of life, bleeding from the stomach appeared, impurities of fresh blood in the feces. Vitamin K and it was not administered in the first day of life. What is the most likely diagnosis to think about?

Гемофілія Hemophilia

Гемолітична хвороба новонароджених Hemolytic disease of the newborn

Коагулопатія при захворюваннях печінки Coagulopathy in liver diseases

Геморагічна хвороба новонароджених Hemorrhagic disease of newborns

Ізоімунна тромбоцитопенія Isoimmune thrombocytopenia

90 / 147
У хворого чоловіка 36 років під час бою куля влучила в передню стінку черева 5 годин тому. Стан тяжкий, AT - 100/60 мм рт. ст., пульс - 90/хв. Живіт напружений, дошкоподібний, різко болючий у всіх відділах. Яке дослідження слід виконати для підтвердження відкритої травми черева з перфорацією порожнинного органу? A sick man, 36 years old, was hit by a bullet in the front wall of the abdomen 5 hours ago during the battle. The condition is serious, AT - 100/60 mm Hg, pulse - 90/min. The abdomen is tense, board-like, sharply painful in all departments. What research should be performed to confirm an open injury of the abdomen with perforation of a cavity organ?

МРТ черевної порожнини MRI of the abdominal cavity

Комп’ютерна томографія Computed tomography

Контрастна рентгенографія шлунково-кишкового шляху Contrast x-ray of the gastrointestinal tract

Оглядова рентгенографія черевної порожнини Overview x-ray of the abdominal cavity

УЗД органів черевної порожнини Ultrasound of abdominal organs

91 / 147
До лікарні за 2 год. після падіння з висоти доставлено хворого чоловіка 45 років у вкрай тяжкому стані. Об’єктивно встановлено: мозкова кома, нижній парапарез, артеріальний тиск - 100/600 мм рт.ст., частота серцевих скорочень - 72/хв., дихання поверхневе, закритий перелом кісток тазу та лівого стегна. До якого відділення слід госпіталізувати хворого? 2 hours after a fall from a height, a 45-year-old man was brought to the hospital in extremely serious condition. Objectively established: cerebral coma, lower paraparesis, blood pressure - 100/600 mm Hg, heart rate - 72/min, shallow breathing, closed fracture of pelvis and left hip. To which department should the patient be hospitalized?

Ортопедичне відділення Orthopedic Department

Відділення травматології Department of Traumatology

Відділення нейрохірургії Department of Neurosurgery

Відділення реанімації ICU

Хірургічне відділення Surgical Department

92 / 147
У вагітної жінки, яка страждає на цукровий діабет, у сірок гестації 38 тижнів, раптово з’явилась загальна слабкість, почуття голоду, тремтіння в тілі, значне потовиділення, порушення свідомості. Об’єктивно виявлено: дихання не порушене, шкіра волога, тонус очних яблук і тонус м’язів нормальний, пульс - 100/хв., АД -110/70 мм рт. ст. Які найперші заходи? A pregnant woman suffering from diabetes, at 38 weeks of gestation, suddenly developed general weakness, hunger, body tremors, profuse sweating, loss of consciousness. Objectively revealed: breathing is not disturbed, skin is moist, tone of eyeballs and muscle tone is normal, pulse - 100/min., blood pressure -110/70 mm Hg. What are the first measures?

Введення адреналіну Injection of adrenaline

Введення глюкагону Glucagon administration

Введення 40% - го розчину глюкози Introduction of 40% glucose solution

Введення інсуліну Insulin administration

Введення 5% - го розчину глюкози Introduction of 5% glucose solution

93 / 147
Хворий чоловік 42 років отримав травму голови під час дорожньо- транспортної пригоди. Короткочасно втратив свідомість. За 4 години після травми стан хворого погіршився. Свідомість за шкалою ком Глазго - 8 балів. Ліва зіниця дещо ширша, ніж права. Який метод обстеження найбільш інформативний у цьому разі? A 42-year-old man suffered a head injury during a traffic accident. He briefly lost consciousness. 4 hours after the injury, the patient's condition worsened. Consciousness according to the Glasgow coma scale - 8 points. The left pupil is slightly wider than the right. Which examination method is the most informative in this case?

Комп’ютерна томографія головного мозку Computed tomography of the brain

Елсктроенцефалографія Elsctroencephalography

Церебральна ангіографія Cerebral angiography

Ехоснцефалографія Echosncephalography

Рентгенографія черепа X-ray skull

94 / 147
У хворого чоловіка 41 року після харчової погрішності виникла гіпертермія до 39°С, нудота, багатокрагна блювота, діарея, слабкість, болі в кістках. Проводилася антибактеріальна та інфу- зіина терапія колоїдами і кристалоїдами. Попри терапію AT знизився до 70/0 мм рт. ст., виражена слабкість, тахікардія до 118/хв. Яка найбільш ефективна лікувальна стратегія у цьому разі? A 41-year-old male patient developed hyperthermia up to 39°C, nausea, multi-collared vomiting, diarrhea, weakness, bone pain after eating a wrong food. Antibacterial and infu- the patient's therapy with colloids and crystalloids. Despite the therapy, AT decreased to 70/0 mm Hg, pronounced weakness, tachycardia up to 118/min. What is the most effective treatment strategy in this case?

Інфузійна терапія + інотропна підтримка Infusion therapy + inotropic support

Інотропна + вазотонічна підтримка Inotropic + vasotonic support

Інфузійна терапія + інотропна + вазотонічна підтримка Infusion therapy + inotropic + vasotonic support

Інфузійна терапія Infusion therapy

Інфузійна терапія + вазотонічна підтримка Infusion therapy + vasotonic support

95 / 147
Хворий чоловік 56 років отримав травму внаслідок ДТП. Скаржиться па біль у лівому стегні і лівій гомілці. Спо-стерігається патологічна рухливість і крепітація в середній третині лівого стегна і лівої гомілки. Яку транспортну іммобілізацію слід застосувати для профілактики травматичного шоку? A sick 56-year-old man was injured in a road accident. He complains of pain in the left thigh and left leg. There is pathological mobility and crepitus in the middle third of the left thigh and left legs. What kind of transport immobilization should be used to prevent traumatic shock?

Шину Дітеріхса Tire of Dieterichs

Імпровізовану іммобілізацію Improvised immobilization

Накласти гіпсову пов’язку Apply a plaster cast

Прибинтувати пошкоджену кінцівку до здорової Try to transplant a damaged limb to a healthy one

Шину Крамера Kramer Tire

96 / 147
Пішохід 54 років під час ожеледиці послизнувся та впав на спину. Стан середньої важкості. Є загальна загальмованість при збереженій свідомості та орієнтуванні, гіперемія обличчя, лока-лізований біль у спині в ділянці грудного відділу хребта. Спостерігається тахікардія, підвищення артеріального тиску. Утруднення дихання не спостерігається. Можна думати про пошкодження грудного відділу хребта. Як ви будете транспортувати хворого? A 54-year-old pedestrian slipped and fell on his back during an icicle. The condition is of medium severity. There is general retardation with preserved consciousness and orientation, facial hyperemia, localized pain in back in the area of ​​the thoracic spine. Tachycardia, increased blood pressure is observed. Difficulty breathing is not observed. You can think about damage to the thoracic spine. How will you transport the patient?

У напівсидячому положенні з палкою в ліктьових згинах In a semi-sitting position with a stick in the elbows

На твердій поверхні на животі On a hard surface on your stomach

На животі на м’яких носилках з використанням шини Еланського On the stomach on a soft stretcher using an Elansky splint

На твердій поверхні на спині On a hard surface on the back

На твердій поверхні з трохи піднятим головним кінцем On a hard surface with the head end slightly raised

97 / 147
На виробництві з виготовлення пластмасових контейнерів сталася аварія. За 1 годину після цього працівник звернувся до медичного пункту зі скаргами на шум у вухах, головний біль, запаморочення, загальну слабкість, підвищення температури тіла до фебрильної, слинотечу та пронос, сірий відтінок шкірних покривів. За півтори години звернулись ще 6 працівників зі схожою клінікою. У всіх постраждалих у крові було виявлено метгемоглобінемію. Яка речовина найімовірніше викликала таке отруєння? An accident occurred at the production of plastic containers. 1 hour after that, the worker went to the medical center with complaints of tinnitus, headache, dizziness, general weakness , an increase in body temperature to febrile, drooling and diarrhea, a gray shade of the skin. In an hour and a half, 6 more workers contacted a similar clinic. Methemoglobinemia was detected in the blood of all the victims. What substance most likely caused such poisoning?

Синильна кислота Pranic acid

Похідні фенолу Phenol derivatives

Монооксид вуглецю Carbon monoxide

Сірководень Sulfur

Аміак Ammonia

98 / 147
З приводу ОРЗ жінка 48 років часто використовує анальгін, аспірин, левоміцетин. Після переохолодження в неї розвинувся гострий гнійний отит. У загальному аналізі крові виявлено: ери-троцити - 4.3 * 1012/л, гемоглобін - 132 г/л, лейкоцити -1,2- 109/л, с - 0%, пал - 1 %, е - 6%, л - 83%, м -10%, тромбоцити - 170 • 109/л, ШОЕ - 48 мм/год. Який стан розвинувся у хворої? A 48-year-old woman often uses analgin, aspirin, and chloramphenicol for acute respiratory infections. After hypothermia, she developed acute purulent otitis. A general blood test revealed: erythrocytes - 4.3 * 1012/l, hemoglobin - 132 g/l, leukocytes -1.2- 109/l, c - 0%, pal - 1%, e - 6%, l - 83%, m -10%, platelets - 170 • 109/l, ESR - 48 mm/h. What condition has developed in the patient?

Агранулоцитоз Agranulocytosis

Хронічна лімфоїдна лейкемія Chronic lymphoid leukemia

Гостра лейкемія Acute leukemia

Лейкемічна реакція Leukemic reaction

Лімфогранулематоз Lymphogranulomatosis

99 / 147
Хворий чоловік 72 років із серцевими болями на тлі пневмонії. Під час транспортування до відділення реанімації раптово втратив свідомість, спостерігається мідріаз, відсутність периферичного пульсу. Невідкладні лікувальні заходи: A sick 72-year-old man with heart pain on the background of pneumonia. During transportation to the intensive care unit, he suddenly lost consciousness, mydriasis is observed, there is no peripheral pulse. Emergency medical measures:

Лідокаїн внутрішньовенно Lidocaine IV

Адреналін внутрішньовенно Adrenaline intravenously

Дефібриляція Defibrillation

Преднізолон внутрішньовенно Prednisone IV

Серцево-легенева реанімація Cardiopulmonary resuscitation

100 / 147
До реанімаційного відділення доставлений хворий чоловік 64 років. Під час огляду встановлено: свідомість відсутня, зіниці розширені, фотореакція відсутня, спонтанне дихання, AT та пульс на сонних артеріях не визначаються, серцеві тони не прослуховуються. На кардіомоніторі реєструються рідкі широкі комплекси QRS. Який із реанімаційних заходів не є найпершим? A 64-year-old sick man was brought to the intensive care unit. During the examination, it was found that: consciousness is absent, the pupils are dilated, there is no photoreaction, spontaneous breathing, AT and pulse on the carotid arteries are not are determined, heart sounds are not heard. Rare wide QRS complexes are registered on the cardiomonitor. Which of the resuscitation measures is not the first?

Електрична дефібриляція Electrical defibrillation

Інтубація трахеї Tracheal intubation

Внутрішньовенне струменеве уведення адреналіну Intravenous jet injection of epinephrine

Штучна вентиляція легень Artificial lung ventilation

Непрямий масаж серця Indirect heart massage

101 / 147
До приймального відділення лікарні з місця події доставлено потерпілого чоловіка 72 років із тяжкою політрав- мою внаслідок дорожньо-транспортної пригоди. Стан постраждалого вкрай тяжкий, спонтанне дихання агональне, гемодинаміка нестабільна. Під час перекладання постраждалого з носилок виникла зупинка кровообігу. Реанімаційні заходи, які потрібно розпочати мають включати штучну вентиляцію легень та непряму компресію міокарда у співвідношенні: A 72-year-old male victim with severe polytrauma as a result of a traffic accident was brought to the hospital admissions department from the scene of the incident. The victim's condition is extremely serious, spontaneous breathing is agony, hemodynamics unstable. Circulatory arrest occurred during transfer of the victim from the stretcher. Resuscitation measures to be initiated should include artificial ventilation of the lungs and indirect myocardial compression in the ratio:

1 1

С 1 C 1

2 2

1 1

102 / 147
У жінки 54 років з’явилися болі та почервоніння за ходом варикозно-розширених вен правої гомілки. Упродовж доби почервоніння та інфільтрація поширилась на нижню третину лівого стегна. Під час пальпації виявлено щільний, болючий інфільтрат за ходом великої підшкірної вени, який пальпується до рівня середньої третини лівого стегна. Яку лікувальну тактику необхі-дно застосувати? A 54-year-old woman developed pain and redness along the course of varicose veins of the right lower leg. Over the course of a day, the redness and infiltration spread to the lower third of the left thigh. During palpation revealed a dense, painful infiltrate along the course of the great saphenous vein, palpable to the level of the middle third of the left thigh. What treatment tactics should be used?

Амбулаторне лікування Outpatient treatment

Ургентна операція Urgent operation

Планова операція Planned operation

Туге еластичне бинтування Tight Elastic Bandage

Антикоагулянтна терапія Anticoagulation therapy

103 / 147
У хлопчика, який був госпіталізований до відділення дитячої інтенсивної терапії після ураження електричним струмом, за 6 годин після електротравми на екрані кардіомонітору зафіксована дрібнохвильова фібриляція шлуночків. Якою повинна бути енергія першого розряду під час виконання електроде- фібриляції? In a boy who was hospitalized in the pediatric intensive care unit after being electrocuted, 6 hours after the electrocution, small-wave ventricular fibrillation was recorded on the heart monitor screen. What should be the energy first discharge during electrodefibrillation?

2 - 4 Дж/кг 2 - 4 J/kg

0,5 -1 Дж/кг 0.5 -1 J/kg

5 - 6 Дж/кг 5 - 6 J/kg

7 - 8 Дж/кг 7 - 8 J/kg

9-10 Дж/кг 9-10 J/kg

104 / 147
Причиною порушення функції нирок під час застосування норадреналіну є: The cause of kidney dysfunction during the use of norepinephrine is:

Спазм артерій і вен мозкового шару нирок Spasm of the arteries and veins of the medulla of the kidneys

Причина ураження нирок не зрозуміла The cause of kidney damage is not clear

Ураження канальців Tubular lesions

Токсична дія на мозковий шар нирок Toxic action on the medulla of the kidneys

Ураження кіркового шару нирок Kidney cortical lesion

105 / 147
У хворого чоловіка 67 років, який перебував на лікуванні у відділенні інтенсивної терапії з приводу тяжкого гіповолемічного шоку, раптово виникла втрата свідомості з подальшою зупинкою серцево-судинної діяльності та дихання: пульс на магістральних судинах не визначається, артеріальний тиск - 0/0 мм рт.ст. Ситуація була розцінена як клінічна смерть, через що почали проводити реанімаційні заходи, які про-довжувалися протягом 45 хвилин, однак марно. Якою має бути подальша тактика лікаря? A 67-year-old male patient, who was being treated in the intensive care unit for severe hypovolemic shock, suddenly lost consciousness followed by cardiovascular and respiratory arrest : the pulse on the main vessels is not determined, the arterial pressure is 0/0 mm Hg. The situation was considered as clinical death, because of which resuscitation measures were started, which continued for 45 minutes, but in vain. What should be the further tactics of the doctor ?

Перевести хворого на штучне дихання Transfer the patient to artificial respiration

Провести дефібриляцію Perform defibrillation

Поставити кардіостимулятор Place a pacemaker

Зупинити реанімаційні заходи Stop resuscitation measures

Продовжувати реанімаційні заходи Continue resuscitation measures

106 / 147
У військовослужбовця, який перебував у зоні бойових дій у ситуації загрози його життю, протягом 1 місяця спостерігається безпомічний вираз обличчя, чоловік дезорієнтований, не пам’ятає свого віку, назви речей. Який попередній діагноз? A serviceman who was in a combat zone in a life-threatening situation has a helpless facial expression for 1 month, the man is disoriented, does not remember his age, names of things. What is the previous diagnosis?

Синдром псевдодеменції Pseudodementia Syndrome

Симулянт Simulant

Сензитивний шизоїд Sensitive Schizoid

Шизофренія Schizophrenia

Психоз Psychosis

107 / 147
Хвора дівчина 22 років, студентка 4-го курсу педагогічного університету. Завжди була замкнута, друзів не мала, проживає в студентському гуртожитку. Два тижня як перестала відвідувати заняття, змінилась у поведінці, за собою не слідкує, не вмивається. Постійно щось пише, до того ж у тексті вичурні символічні знаки. Заявляла, що вона матір божа, повинна врятувати людей від світової війни, вона розмовляє з Нострадамусом. Останні дві доби не спить, заявила, що повинна себе принести в жертву. Викликали дільничного психіатра. Які його дії? A sick girl, 22 years old, a student of the 4th year of a pedagogical university. She was always withdrawn, had no friends, lives in a student dormitory. Two weeks after she stopped attending classes, she has changed in her behavior, she does not take care of herself, she does not wash herself. She is constantly writing something, besides, there are fancy symbolic signs in the text. She declared that she is the mother of God, she must save people from the world war, she talks to Nostradamus. She has not slept for the last two days, declared that she had to sacrifice herself. They called the district psychiatrist. What are his actions?

До сімейного лікаря To the family doctor

До жіночої консультації за місцем проживання To the women's consultation at the place of residence

До місцевої поліції To the local police

На судово-медичну експертизу For forensic examination

До психіатра To the psychiatrist

108 / 147
Хворий чоловік 86 років перебуває в стаціонарі хірургічного відділення після ампутації нижньої кінцівки на рівні стегна з приводу діабетичної стопи. За добу після оперативного втручання стан хворого почав раптово прогресивно погіршуватись. Порушилась свідомість: помірне оглушення, виникло часте поверхневе дихання, хворий став блідим, з’явився холодний піт. На ЕКГ на час прибуття, спостерігається: синусова тахікардія з ЧСС-148/хв., блокада правої ніжки пучка Гісса, високий загострений зубець Р, ознаки гіпертрофії правого шлуночка. Хірург запідозрив ТЕЛА і викликав реаніматолога. На час огляду реаніматологом виявлено: свідомість на рівні коми, пульс на магістральних судинах відсутній, дихання відсутнє. Які найперші дії має застосувати реаніматолог? An 86-year-old man is hospitalized in the surgical department after amputation of the lower extremity at the level of the thigh due to diabetic foot. A day after the operation, the patient's condition began to deteriorate suddenly and progressively. Consciousness was disturbed: moderate stupor, frequent shallow breathing occurred, the patient became pale, cold sweat appeared. On the EKG at the time of arrival, it is observed: sinus tachycardia with a heart rate of 148/min., blockade of the right leg of the bundle of His, a high pointed P wave, signs of right ventricular hypertrophy. The surgeon suspected PE and called the intensivist. During the examination by the intensivist, it was found: consciousness at the level of coma, no pulse on the main vessels, no breathing. What are the first actions the intensivist should take?

Серцево-легенева реанімація Cardiopulmonary resuscitation

Електрична дефібриляція Electrical defibrillation

Оксигенотерапія Oxygenotherapy

В/в введення гепарину IV heparin

В/в введення метопрололу IV administration of metoprolol

109 / 147
У чоловіка під час їжі розвинулася аспірація стороннього тіла з повною обструкцією дихальних шляхів. Прийом Геймліха неефективний. Значно наріс ціаноз, хворий втратив свідомісгь. Що необхідно зробити насамперед для врятування його життя? A man developed aspiration of a foreign body while eating with complete obstruction of the respiratory tract. Heimlich administration is ineffective. Cyanosis increased significantly, the patient lost consciousness. What should be done first of all to save his life?

Трахеотомію Tracheotomy

Пальцеву ревізію ротової порожнини Finger inspection of the oral cavity

Конікотомію Conicotomy

Введення спазмолітиків Introduction of antispasmodics

Потрійний прийом Сафара Triple reception of Safar

110 / 147
Ви лікар швидкої допомоги, приїхали на виклик. На вулиці лежить людина без свідомості. Температура повітря - 20° С, вітер. Під час огляду постраждалого спостерігається різка блідість шкіри, кистей, шкіра холодна на дотик, пальці покриті шаром льоду. Які Ваші Дії? You are an ambulance doctor, you have arrived on call. There is an unconscious person lying on the street. The air temperature is 20° C, windy. During the examination of the victim, a sharp paleness of the skin is observed , hands, the skin is cold to the touch, the fingers are covered with a layer of ice. What are your actions?

Пов’язки не накладати і доставити хворого до опікового центру Do not apply bandages and take the patient to the burn center

Пов’язки не накладати і доставити хворого до терапевтичного відділення Do not apply bandages and deliver the patient to the therapeutic department

Накласти теплоізолюючі пов’язки і доставити хворого до стаціонару Apply heat-insulating bandages and take the patient to the hospital

Накласти мазеві пов’язки і доставити до реанімаційного відділення Apply ointment bandages and deliver to the intensive care unit

Накласти пов’язки з антисептиком і доставити до стаціонару Apply bandages with an antiseptic and deliver to a hospital

111 / 147
Дитина 16 років непритомна. Зі слів випадкових перехожих втратила свідомість раптово. Шкіра та слизові оболонки ціанотичні. Дихання не вислуховується. Пульс па магістральних артеріях не визначається. Яка невідкладна допомога? A 16-year-old child is unconscious. According to bystanders, she lost consciousness suddenly. The skin and mucous membranes are cyanotic. Breathing is not heard. The pulse on the main arteries is not determined. What is the emergency care ?

Розпочати серцево-легеневу реанімацію Start CPR

Викликати бригаду швидкої допомоги Call an ambulance

Почати внутрішньовенне введення ліків Start IV administration

Покликати на допомогу Call for help

Провести рефлекторні проби Conduct reflex tests

112 / 147
Хворий чоловік 36 років скаржиться на біль за грудиною, задишку в спокої, слабкість. Переніс пневмонію. Об’єктивно встановлено: шкіра ціанотична, розширення вен шиї, грудної стінки, AT - 75/60 мм рт.ст., пульс - 124/хв., слабкого наповнення. Тони серця різко ослаблені, шуми відсутні. Перкуторно межі серця не розширені. Верхівковий поштовх не визначається. Печінка збільшена. На ЕКГ виявлено зниження вольтажу, деформація зубців Т, депресія S-Т. Рентгенологічно виявлено: тінь серця трапецієвидна, різко розширена, талія відсутня, пульсація не простежується. Оберіть першочерговий лікувальний захід: A 36-year-old male patient complains of pain behind the sternum, shortness of breath at rest, weakness. He suffered pneumonia. Objectively established: cyanotic skin, dilatation of veins of the neck, chest wall , AT - 75/60 ​​mm Hg, pulse - 124/min, weak filling. Heart sounds are sharply weakened, there are no murmurs. Percussion, the borders of the heart are not expanded. The apical impulse is not determined. The liver is enlarged. A decrease in voltage is detected on the ECG, deformation of T waves, S-T depression. X-ray revealed: the shadow of the heart is trapezoidal, sharply expanded, the waist is absent, pulsation is not traceable. Choose the primary treatment measure:

Відкрита перикардіотомія Open pericardiotomy

Перикардектомія Pericardectomy

Медикаментозне лікування Medical treatment

Пункція перикарду Pericard puncture

Закрита перикардіотомія Closed pericardiotomy

113 / 147
Чоловік 40 років доставлений до приймального відділення в алкогольній комі. Після проведення дезінтоксикації прийшов до тями, проте надалі стан почав погіршуватися, з’явилися задишка, психомоторне збудження, тахікардія, гіпертензія. У ЗАК зазначено: лейкоцитоз з паличкоядерним зсувом. Яке обстеження треба зробити насамперед для з’ясування причини погіршення ста-ну? A 40-year-old man was brought to the reception department in an alcoholic coma. After detoxification, he regained consciousness, but later his condition began to deteriorate, shortness of breath, psychomotor agitation, tachycardia appeared , hypertension. In the ZAC it is indicated: leukocytosis with a rod-nuclear shift. What examination should be done first of all to find out the cause of the deterioration of the condition?

Рентгенографію кісток черепа X-ray of skull bones

Визначення вмісту глюкози в крові Determination of blood glucose

Рентгенографію органів грудної клітки X-ray of chest organs

Люмбальну пункцію Lumbar puncture

Загальний аналіз сечі General urinalysis

114 / 147
У вагітної жінки 18 років, у якої в анамнезі зазначено самовільний викидень, скарги на рясні кров’янисті виділення зі статевих шляхів, переймоподібні болі внизу живота. AT - 100/60 мм рт.ст., пульс - 96/хв. Остання менструація 2,5 місяці тому. Бімануально встановлено: цервікальний канал пропускає палець. Матка збільшена до 6 тижнів вагітності, не болюча. Жінка наполягає на збереженні вагітності. Якою повинна бути лікувальна тактика? An 18-year-old pregnant woman with a history of spontaneous abortion, complaints of profuse bleeding from the genital tract, spasm-like pains in the lower abdomen. AT - 100 /60 mm Hg, pulse - 96/min. Last menstruation 2.5 months ago. Bimanually established: the cervical canal passes a finger. The uterus is enlarged to 6 weeks of pregnancy, not painful. The woman insists on preserving the pregnancy. What should be treated tactics?

Седативна терапія Sedation therapy

Вичікуюча тактика Waiting tactics

Спазмолітична терапія Spasmolytic therapy

Накладання шва на шийку матки Putting a seam on the cervix

Вишкрібання стінок порожнини матки Scraping the walls of the uterine cavity

115 / 147
Дівчинку 3 років доставлено до приймального відділення каретою ШМД. Під час огляду стан дитини дуже важкий. В свідомості. Менінгеальних симптомів немає. Температура тіла - 40,0°С, кінцівки холодні. Виражені розлади мікроциркуляції. На шкірі нижніх кінцівок, сідниць, тулуба, обличчя - зливна “зірчаста” геморагічна висипка, що швидко розповсюджується. Невеличкі крововиливи на слизових оболонках. Висипка з’явилась декілька годин тому. ЧД - 48/хв., ЧСС - 160/хв. AT - 55/30 мм рт. ст. Який імовірний діагноз? A 3-year-old girl was brought to the reception department by ambulance. During the examination, the child's condition is very serious. She is conscious. There are no meningeal symptoms. Body temperature - 40.0°C , the limbs are cold. Pronounced microcirculation disorders. On the skin of the lower limbs, buttocks, trunk, face - a confluent 'star' hemorrhagic rash that quickly spreads. Small hemorrhages on the mucous membranes. The rash appeared a few hours ago. BH - 48/min. , heart rate - 160/min. AT - 55/30 mm Hg. What is the probable diagnosis?

Тромбоцитопенічна пурпура Thrombocytopenic purpura

Блискавична менінгококцемія Bulky meningococcemia

Геморагічна лихоманка Hemorrhagic fever

Геморагічний васкуліт Hemorrhagic vasculitis

Аддісонічний криз Addisonian crisis

116 / 147
У 50-річного чоловіка протягом 2 днів спостерігаються рідкі випорожнення чорного кольору. Болю у епігастральній ділянці немає. Під час огляду виявлено: блідість шкірних покривів і кон’юнктиви, “судинні зірочки” на шкірі грудей, розширення вен передньої стінки живота, гепатомегалія. Артеріальний тиск - 105/60 мм рт. ст. Промивні води шлунка типу “кавової гущі’.’ Лікар визначив кровотечу з вен стоавоходу. Які препарати разом із трансфузійною та гемостатичною терапією слід призначити насамперед? A 50-year-old man has had loose black stools for 2 days. There is no pain in the epigastric region. During the examination, the following were found: pallor of the skin and conjunctiva, 'vascular stars' on the skin of the chest, dilatation of the veins of the anterior abdominal wall, hepatomegaly. Blood pressure - 105/60 mm Hg. Gastric lavage of the 'coffee grounds' type. The doctor identified bleeding from the veins of the shunt. What drugs along with transfusion and hemostatic therapy should be prescribed first?

Аргініну глутамат Arginine glutamate

Вазопресин Vasopressin

Тіотриазолін Thiotriazoline

Мстоклонрамід Mstoclonramide

Алюмінію гідрооксид Aluminum hydroxide

117 / 147
Жінку 40 років у ділянку нижньої губи вжалила бджола, за декілька хвилин з’явився сильний зуд та набряк нижньої губи, що розповсюдився на обличчя. Який препарат необхідно ввести негайно? A 40-year-old woman was stung by a bee in the area of ​​the lower lip, after a few minutes severe itching and swelling of the lower lip appeared, which spread to the face. What drug should be administered immediately ?

Лінкоміцин Lincomycin

Папаверину гідрохлорид Papaverine hydrochloride

Сульфокамфокаїн Sulfocamphocaine

Атропіну сульфат Atropine sulfate

Дексаметазон Dexamethasone

118 / 147
У приймальному відділені надається невідкладна допомога потерпілому чоловіку з травмою шийного відділу хребта в стані спінального шоку та порушенням зовнішнього дихання. Які із запропонованих реанімаційних заходів не рекомендовано проводити під час цієї патології? In the reception department, emergency care is provided to a male victim with a cervical spine injury in a state of spinal shock and impaired external breathing. Which of the proposed resuscitation measures are not recommended to be performed during this pathologies?

Прийом Сафара під час відновлення прохідності дихальних шляхів Admission of Safar during restoration of airway patency

Внутрішньовенну інфузію плазмозамінників Intravenous infusion of plasma substitutes

Інтубацію трахеї Tracheal intubation

Штучну вентиляцію легень Artificial lung ventilation

Внутрішньовенну інфузію дофаміна Intravenous infusion of dopamine

119 / 147
Молода жінка 30 років, яка страждає на ревматичну мітральну ваду серця з порушенням серцевого ритму (миготлива аритмія), раптово відчула різкий біль у правій руці, оніміння, похоло-дання передпліччя і кисті. Під час огляду встановлено, що шкіра правої верхньої кінцівки бліда, холодна. Активні рухи в суглобах порушені. Пульсація плечової артерії обривається в середній третині плеча. Яка патологія має місце у хворої? A 30-year-old young woman suffering from rheumatic mitral heart disease with irregular heart rhythm (atrial fibrillation) suddenly felt a sharp pain in her right arm, numbness, coldness examination of the forearm and hand. During the examination, it was found that the skin of the right upper limb is pale and cold. Active movements in the joints are impaired. Pulsation of the brachial artery stops in the middle third of the shoulder. What pathology is present in the patient?

Флеботромбоз правої верхньої кінцівки Phlebothrombosis of the right upper extremity

Облітеруючий тромбангіїт Thrombangiitis obliterans

Емболія правої плечової артерії Embolism of the right brachial artery

Хвороба Рейно Raynaud's disease

Артеріїт правої плечової артерії Arteritis of the right brachial artery

120 / 147
У сержанта 30 років після вибуху авіабомби ліва гомілка була притиснута бетонною стіною до землі. Звільнення від стиснення гомілки відбулося за 3 години, потерпілого доставлено до шпиталю. Шкіра гомілки блідо-ціанотична, холодна на дотик, набрякла. Тони серця приглушені, артеріальний тиск - 80/50 мм рт. ст., пульс слабкого наповнення, 120/хв. У сечі виявлено: міоглобін, протеїнурія та циліндрурія, колір - бурий. Який найбільш імовірний діагноз? 30 years after the explosion of an aerial bomb, the sergeant's left leg was pressed against the ground by a concrete wall. The leg was released from compression in 3 hours, the victim was taken to the hospital. The skin of the leg is pale -cyanotic, cold to the touch, swollen. Heart sounds are muffled, blood pressure - 80/50 mm Hg, weak pulse, 120/min. In the urine: myoglobin, proteinuria and cylinduria, color - brown. What is the most likely diagnosis?

Синдром тривалого стиснення, ранній період Prolonged compression syndrome, early period

Гостра печінкова недостатність Acute liver failure

Гострий гломерулонефрит Acute glomerulonephritis

Синдром тривалого стиснення, проміжний період Prolonged compression syndrome, intermediate period

Гострий венозний тромбоз Acute venous thrombosis

121 / 147
У хворого чоловіка 55 років під час фізичного навантаження з’явився різкий головний біль у потилиці, блювота, запаморочення. Об’єктивно виявлено: свідомість збережена, пульс ритмічний - 62/хв, артеріальний тиск - 130/90 мм рт. ст. Парезів немає, позитивні менінгеальні симптоми. Який діагноз найбільш ймовірний? A sick 55-year-old man developed a sharp headache in the back of the head during physical exertion, vomiting, dizziness. Objectively revealed: consciousness is preserved, the pulse is rhythmic - 62/min, blood pressure - 130/90 mm Hg. There are no paresis, positive meningeal symptoms. What is the most likely diagnosis?

Субарахноїдальний крововилив Subarachnoid hemorrhage

Менінгіт Meningitis

Гіпертонічний криз Hypertensive crisis

Транзиторна ішемічна атака Transient ischemic attack

Ішемічний інсульт Ischemic stroke

122 / 147
Сімейний лікар оглядає вдома хворого хлопця, у якого протягом 3 днів спостерігаються болі в горлі, слабкість, підвищення температури. Під час огляду виявлено: температура тіла - 38°С, шкіра бліда. Піднебінні мигдалики збільшені, гіперемійовані, із ціанотичним відтінком, на їх поверхні сірувато-білі плівки, не знімаються. Який Ваш попередній діагноз? The family doctor examines a sick boy at home, who has had a sore throat, weakness, and fever for 3 days. During the examination, it was found: body temperature - 38°С , the skin is pale. The palatine tonsils are enlarged, hyperemic, with a cyanotic hue, grayish-white films on their surface, not removed. What is your previous diagnosis?

Аденовірусна інфекція Adenovirus infection

Виконав ручне зміщення вагітної матки вліво Performed a manual displacement of the pregnant uterus to the left

Виконав ручні компресії дна вагітної матки Performed manual compressions of the bottom of the pregnant uterus

Увів міорслаксанти довенно Introduced miorslaxants permanently

Перевів пацієнтку у положення Тренделенбурга Moved the patient into the Trendelenburg position

Виконав підкладання валика під лівий бік Performed laying of the roller under the left side

123 / 147
Чоловік 45 років скаржиться на появу розрідженого калу чорного кольору, загальну слабкість. Упродовж тривалого часу безконтрольно приймає нестероїдні протизапальні препарати з приводу болю у спині. Які дії є найпершими? A 45-year-old man complains of the appearance of thin, black stools, general weakness. He has been taking non-steroidal anti-inflammatory drugs for a long time uncontrollably for back pain. What are the first steps?

Гематокрит Hematocrit

Ректальне обстеження Rectal examination

Рентгенографія шлунка Stomach X-ray

Загальний аналіз крові General blood test

Езофагогастродуодсноскопія Esophagogastroduodenoscopy

124 / 147
Хлопчик 5 місяців доставлений до відділення інтенсивної терапії у стані важкого зневоднення. Хворіє протягом трьох днів - рідкий водянистий стілець 10 - 12 разів на добу, блювання. Під час огляду встановлено: шкіра збирається в складку, велике джерельце та очі дуже запалі, слизові оболонки сухі, різко позитивний симптом “блідої плями’,’ AT - 65/40 мм рт.ст., ЧСС - 178/хв., ЧД - 62/хв. Оберіть стартову інфузійну терапію із запропонованих варіантів: A 5-month-old boy was brought to the intensive care unit in a state of severe dehydration. He has been sick for three days - liquid watery stool 10-12 times a day, vomiting. During the examination established: the skin gathers in a fold, the large fontanel and eyes are very inflamed, the mucous membranes are dry, a sharply positive symptom of the 'pale spot', AT - 65/40 mm Hg, heart rate - 178/min, BH - 62/min Choose the initial infusion therapy from the options offered:

10% - й розчин глюкози 20 мл/кг за годину 10% glucose solution 20 ml/kg per hour

5% - й розчин глюкози 20 мл/кг за годину 5% glucose solution 20 ml/kg per hour

6% - й розчин реополіглюкіну 20 мл/кг за годину 6% rheopolyglucin solution 20 ml/kg per hour

0,9% - й розчин натрія хлориду 20 мл/кг за годину 0.9% sodium chloride solution 20 ml/kg per hour

6% - й розчин гідроетилкрохмалю 20 мл/кг за годину 6% hydroethyl starch solution 20 ml/kg per hour

125 / 147
До відділення травматології доставлено чоловіка 44 років зі зламом кісток тазу. За кілька годин він почав скаржитися на відсутність сечовипускання за наявності бажання це зробити та біль у надлобковій зоні. Під час огляду виявлено повний сечовий міхур, під час ректального обстеження - зсовувана вгору високо “сидяча” простата. Попередній діагноз - розрив уретри. Яку невідкладну допомогу треба надати? A 44-year-old man was brought to the trauma department with a fracture of the pelvic bones. After a few hours, he began to complain about the lack of urination when he wanted to do so and pain in the suprapubic area. Under during the examination, a full bladder was found, during the rectal examination - a high 'sitting' prostate shifted upwards. The preliminary diagnosis is a rupture of the urethra. What emergency care should be provided?

Катетеризація уретри Urethra catheterization

Надлобкова пункція сечового міхура Suprapubic bladder puncture

Знеболююча терапія Pain therapy

Спазмолітична терапія Spasmolytic therapy

Протизапальна терапія Anti-inflammatory therapy

126 / 147
Військовослужбовець доставлений до МПП із зони хімічного ураження. Скаржиться на задишку, кашель із пінистим рожевим харкотинням, свербіж у горлі. Акроціаноз, ЧД - 35/хв„ перкуторний звук коробковий, у нижніх відділах легень гучні дрібно-пухирчасті вологі хрипи. ЧСС - 125/хв. До якої групи БОР відноситься використана хімічна речовина? A military serviceman was brought to the hospital from the area of ​​chemical damage. He complains of shortness of breath, cough with foamy pink sputum, itching in the throat. Acrocyanosis, BH - 35/min' percussive sound boxy, in the lower parts of the lungs there are loud, small, bubbling wet rales. Heart rate - 125/min. To which group of BOR does the used chemical substance belong?

Задушливої дії Suffocating action

Нервово-паралітичної дії Nerve-paralytic action

Психоміметичної дії Psychomimetic action

Шкірно-наривної дії Skin-abscess action

Сльозоточивої дії Lacrimal action

127 / 147
До хірургічного відділення доставлено хворого чоловіка 52 років після дорожньо-транспортної пригоди з підозрою на розрив печінки, внутрішню кровотечу. Які обстеження треба викопати для уточнення діагнозу? A sick 52-year-old man was brought to the surgical department after a traffic accident with suspicion of rupture of the liver, internal bleeding. What examinations should be done to clarify the diagnosis?

Колоноскопію Colonoscopy

Іригоскопію Irigoscopy

УЗД Ultrasound

Лапароцентез Laparocentesis

Оглядову рентгенографію органів черевної порожнини Surveillance x-ray of abdominal organs

128 / 147
До клініки доставлено хворого чоловіка: кома, рухове збудження, тремор, шкірні покриви бліді, вологи. Тони серця ритмічні. AT - 90/70 мм рт. ст., пульс - 108/хв,, центральний венозний тиск - 60 мм вод. ст., дихання - 20/хв. pH 7,4, глікемія - 2,0 ммоль/л. В анамнезі вказано амбулаторне лікування цукрового діабету. Який діагноз? A sick man was brought to the clinic: coma, motor agitation, tremors, skin is pale, wet. Heart sounds are rhythmic. AT - 90/70 mm Hg. , pulse - 108/min,, central venous pressure - 60 mm Hg, breathing - 20/min, pH 7.4, glycemia - 2.0 mmol/l. Ambulatory diabetes treatment is indicated in the anamnesis. What is the diagnosis?

Емболія легеневої артерії Pulmonary embolism

Гіперлактацидемічна кома Hyperlactacidemic coma

Гіповолемічний шок Hypovolemic shock

Інфаркт міокарда Myocardial infarction

Гіпоглікемічна кома Hypoglycemic coma

129 / 147
У 12-річної дівчинки після вживання двох таблеток аспірину температура тіла підвищилася до 39 - 40,0°С. Скаржиться на загальне нездужання, запаморочення, несподівану появу червоних плям на шкірі з утворенням пухирів та відшаровуванням епідермісу з ерозуванням поверхні. Ураження на шкірі нагадують опіки другого ступеню. Симптом Нікольського - позитивний. Який найбільш імовірний діагноз? A 12-year-old girl's body temperature rose to 39-40.0°C after taking two aspirin tablets. She complains of general malaise, dizziness, sudden appearance of red spots on the skin with the formation of blisters and peeling of the epidermis with erosion of the surface. Skin lesions resemble second-degree burns. Nikolsky's symptom is positive. What is the most likely diagnosis?

Багатоформна ексудативна еритема Multiform exudative erythema

Гострий епідермальний некроліз Acute epidermal necrolysis

Еритродермія Erythroderma

Токсикодермія Toxicoderma

Вульгарна пухирчатка Vulgar pemphigus

130 / 147
Під час обстеження лікарем хворого, доставленого з місця отримання ним травми за пів години, установлено: на лівій нижній кінцівці накладено джгут, дистальний відділ кінцівки блідий, хворий неспокійний, шкіра та слизові оболонки бліді, сухість ротової порожнини, периферійні вени запалі, симптом “порожніх судин’’, тахікардія, пульс 110-120/хв., AT - 80/50 мм рт. ст. В аналізі крові виявлено: гемоглобін - 125 г/л, еритроцити - 3,5 • 1012/л. Шоковий індекс (по Альговеру) - 1,5 (дефіцит ОЦК = 30%). Які невідкладні лікувальні дії повинен здійснити лікар-хірург? During the doctor's examination of the patient, who was brought from the place of injury in half an hour, it was established: a tourniquet was applied to the left lower limb, the distal part of the limb was pale, the patient was restless, skin and mucous membranes are pale, dry mouth, inflamed peripheral veins, symptom of 'empty vessels', tachycardia, pulse 110-120/min., AT - 80/50 mm Hg. Blood analysis revealed: hemoglobin - 125 g /l, erythrocytes - 3.5 • 1012/l. Shock index (according to Algover) - 1.5 (BCC deficiency = 30%). What urgent medical actions should the surgeon perform?

Терміново ввести 1 л свіжозамороженої плазми Urgently inject 1 liter of fresh frozen plasma

Перелити хворому еритроцитарну масу і оперувати Transfuse the patient with erythrocyte mass and operate

Провести протишокову терапію, знеболити, після чого хворого оперувати Conduct shock therapy, anesthetize, and then operate on the patient

Зняти джгут та перев’язати травмовані судини Remove the tourniquet and bandage the injured vessels

Інфузійна терапія (сольові або колоїдні розчини, плазмозаміпники), потім оперувати Infusion therapy (saline or colloidal solutions, plasma-entraining agents), then operate

131 / 147
У жінки 30 років внаслідок других пологів народилася дитина з анемічно- жовтяничною формою гемолітичної хвороби. Група крові у жінки А (II) Rh (-), група крові в новонародженого В (III) Rh (+), у батька новонародженого також В (III) Rh (+). Яка найбільш вірогідна причина імуноконфлікту? A 30-year-old woman gave birth to a child with the anemic-jaundic form of hemolytic disease as a result of the second delivery. The woman's blood group is A (II) Rh (-), blood group in the newborn is B (III) Rh (+), the father of the newborn is also B (III) Rh (+). What is the most likely cause of immunoconflict?

Конфлікт по антигену АВ Conflict on antigen AB

Конфлікт по антигену В Conflict on antigen B

Конфлікт по антигену А Conflict on antigen A

Конфлікт по АВ0 Conflict on AB0

Резус-конфлікт Rhesus conflict

132 / 147
Хворий чоловік 40 років скаржиться на біль у правій здухвинній ділянці, нудоту, підвищення температури до 37,2°С. Хворіє 6-й день. Спочатку біль з’явився в епігастральній ділянці, за добу біль локалізувався у правій здухвинній ділянці. Під час огляду виявлено: температура нормальна, напруження м’язів немає. Позитивні симптоми Ровзінга та Сітковського. Симптом Щоткіна-Блюмберга негативний. Який Ваш діагноз? A 40-year-old sick man complains of pain in the right iliac region, nausea, temperature rise to 37.2°C. He has been ill for the 6th day. At first, pain from appeared in the epigastric region, within a day the pain was localized in the right iliac region. During the examination, it was found: the temperature is normal, there is no muscle tension. Positive Rovzing and Sitkovsky symptoms. The Shttkin-Blumberg symptom is negative. What is your diagnosis?

Гострий простатит Acute prostatitis

Кишкова непрохідність Intestinal obstruction

Правосторонній паранефрит Right-sided paranephritis

Гострий апендицит (ретроцекальне розташування) Acute appendicitis (retrocecal location)

Хвороба крона Crohn's disease

133 / 147
У 4-місячного хлопчика повторюються напади різкого неспокою, які супроводжуються приведенням ніжок до живота, блюванням, появою у випорожненнях домішок крові схожих на “смородинове желе” та слизу. Напади тривають 15-30 хвилин, у перервах між якими дитина спить. Який найбільш ймовірний діагноз? A 4-month-old boy has repeated bouts of severe restlessness, which are accompanied by bringing his legs to his stomach, vomiting, the appearance of blood impurities similar to 'currant jelly' and mucus in his stools. Attacks last 15-30 minutes, in between which the child sleeps. What is the most likely diagnosis?

Гострий гастроентероколіт Acute gastroenterocolitis

Атрезія жовчовивідних шляхів Biliary atresia

Інвагінація кишечника Intussusception

Гострий гепатит Acute hepatitis

Гострий апендицит Acute appendicitis

134 / 147
4-річна дитина доставлена до приймального відділення. Мати звернула увагу, що дитина гралась таблетками парацетамолу і кілька таблеток у конвалюті не вистачає. З моменту прийому таблеток, орієнтовно, пройшло 40 хвилин. Що повинен зробити лікар насамперед? 4-year-old child was taken to the reception department. The mother noticed that the child was playing with paracetamol tablets and several tablets were missing from the money box. It has been roughly since taking the tablets 40 minutes. What should the doctor do first?

Провести форсований діурез Perform forced diuresis

Стимулювати блювоту Stimulate vomiting

Призначити активоване вугілля Assign activated carbon

Промити шлунок Wash the stomach

Призначити сольовий проносний Prescribe saline laxative

135 / 147
У коридорі лікарні хворому чоловіку 55 років раптом стало погано. Його негайно оглянув лікар. Під час огляду виявлено: шкірні покриви бліді, самостійне дихання відсутнє, пульсу на сонних артеріях немає. Який із запропонованих заходів реанімації слід вжити насамперед? In the corridor of the hospital, a sick 55-year-old man suddenly felt sick. He was immediately examined by a doctor. During the examination, it was found: pale skin, no independent breathing, pulse on the carotid arteries no. Which of the proposed resuscitation measures should be taken first?

Непрямий масаж серця Indirect heart massage

В/в введення атропіну IV administration of atropine

В/в введення адреналіну IV administration of epinephrine

Дихання ”рот до роту” Mouth to mouth breathing

Дефібриляція Defibrillation

136 / 147
Дитина народилася у терміні 43 тижня, маса тіла - 4100 г, низький грубий голос, набряки на тильній поверхні кистей, пупкова грижа, пролонгована жовтяниця 200 мкмоль/л на 10 - й день життя, закрепи. Діагностовано вроджений гіпотиреоз. Яку невідкладну допомогу потрібно надати? The child was born at 43 weeks, body weight - 4100 g, low rough voice, edema on the back surface of the hands, umbilical hernia, prolonged jaundice 200 μmol/l on 10th day of life, constipation. Congenital hypothyroidism is diagnosed. What emergency care should be provided?

Призначення сечогінних препаратів Prescription of diuretics

Призначення еубіотиків, лактулози Prescription of eubiotics, lactulose

Проведення фототерапії Carrying out phototherapy

Лікування L тироксином L thyroxine treatment

Хірургічна корекція пупкової грижі Surgical correction of umbilical hernia

137 / 147
У хворого чоловіка 50 років після надмірного вживання алкоголю раптово виник інтенсивний біль в епігастрії. Пульс - 100/хв., екстрасистолія. АТ- 90/60 мм рт. ст. Тони серця різко осла-блені. Лейкоцити -13,8- 109/л. На ЕКГ виявлено: сегмент ST у відведеннях II, III, aVF вище ізолінії на З мм. Яка тактика лікаря на догоспітальному етапі? A sick 50-year-old man suddenly developed intense pain in the epigastrium after excessive alcohol consumption. Pulse - 100/min., extrasystole. Blood pressure - 90/60 mm Hg. Art. Heart tones are sharply weakened. Leukocytes -13.8-109/l. The ECG revealed: ST segment in leads II, III, aVF above the isoline by 3 mm. What are the doctor's tactics at the prehospital stage?

Промивання шлунку Gastric lavage

Введення анальгетиків, виклик дільничного лікаря Introduction of analgesics, calling the district doctor

Введення реосорбілакту, анальгетиків Introduction of reosorbilact, analgesics

Введення кордарону Cordaron input

Уведення анальгетиків, аспірин, госпіталізація у BAIT Administration of analgesics, aspirin, hospitalization in BAIT

138 / 147
Пацієнтка 48 років з електротравмою II ступеня та електроопіками правої верхньої кінцівки ШБ ст. 2% п.т. Анамнез достовірно не відомо - мала місце втрата свідомості. Яке інструментальне дослідження є обов’язковим у разі електротравмі? A 48-year-old female patient with II-degree electrocution and electric burns of the right upper extremity of the right upper extremity. 2% p.t. Anamnesis is not reliably known - there was a loss of consciousness. What instrumental is research mandatory in case of electrocution?

МРТ MRI

УЗД внутрішніх органів Ultrasound of internal organs

Електрокардіографія Electrocardiography

Рентгенографія уражених кінцівок X-ray of affected limbs

УЗД судин уражених кінцівок Ultrasound of vessels of affected limbs

139 / 147
Під час аварії на атомній електростанції черговий персонал та деякі пожежники отримали дозу радіоактивного опромінення 4 - 6 Гр. У кажіть форму ГПХ, яка розвинеться в постраждалих: During the accident at the nuclear power plant, the personnel on duty and some firefighters received a dose of radioactive radiation of 4 - 6 Gy. Indicate the form of GPC that will develop in the victims:

Гостра променева реакція Acute radiation reaction

Кишкова Intestinal

Кістково-мозкова Bone brain

Судинно-токсемічна Vascular-toxemic

Церебральна Cerebral

140 / 147
Чоловік 39 років скаржиться на підвищення температури тіла до 39°С, озноб, профузне потовиділення, тиждень тому було переохолодження. Симптом Пастернацького різко позитивний з обох сторін. Дані УЗД: збільшені розміри обох нирок. Аналіз крові: лейкоцити - 12 • 109/л, UIOE - 25 мм/год, е - 2, п - 15, с - 45, л - ЗО, м - 8. Аналіз сечі: питома вага - 1015, білок - 0,035 г/л, лейкоцити - вкривають все п/з, еритроцити - 8 -10 в п/з, слиз +++, бактерії +++. Поставте діагноз: A 39-year-old man complains of an increase in body temperature to 39°C, chills, profuse sweating, hypothermia a week ago. Pasternacki's symptom is sharply positive on both sides. Ultrasound data : increased size of both kidneys. Blood analysis: leukocytes - 12 • 109/l, UIOE - 25 mm/h, e - 2, p - 15, c - 45, l - ZO, m - 8. Urine analysis: specific gravity - 1015, protein - 0.035 g/l, leukocytes - cover all p/z, erythrocytes - 8-10 in p/z, mucus +++, bacteria +++. Make a diagnosis:

Хронічний пієлонефрит Chronic pyelonephritis

Гострий двобічний пієлонефрит Acute bilateral pyelonephritis

Гострий двобічний гломерулонефрит Acute bilateral glomerulonephritis

Підгострив гломерулонефрит Aggravated glomerulonephritis

Амілоїдоз нирок Kidney amyloidosis

141 / 147
У породіллі в ранній післяпологовий період виникла гіпотонічна кровотеча, що продовжується. Загальна крововтрата склала 15% від ОЦК. Об’єктивно встановлено: шкірні покрови бліді, акроціаноз, жінка адинамічна, пульс - 130/хв., AT - 70/50 мм рт. ст., олі- гурія. Яка невідкладна допомога? In the early postpartum period, hypotonic bleeding occurred in a woman in labor, which continues. The total blood loss was 15% of the BCC. Objectively established: pale skin, acrocyanosis, woman adynamic, pulse - 130/min., AT - 70/50 mm Hg, oliguria. What is the emergency care?

Накласти затискачі на шийку матки Put clamps on the cervix

Екстирпація матки Uterus extirpation

Перев’язка внутрішньої клубової артерії Internal iliac artery ligation

Перев’язка маткових судин Ligation of uterine vessels

Надпіхвова ампутація матки Supravaginal amputation of the uterus

142 / 147
Потерпілий чоловік працює на хімічному підприємстві з виробництва азотної кислоти. Скаржиться на сльозотечу, нежить, відчуття першіння в горлі. За 12 годин з’явилась задишка, відчуття нестачі повітря. Об’єктивно встановлено: положення хворого - напівсидяче, шкірні покриви ціанотичні. Під час аускультації - сухі та вологі хрипи над всією поверхнею легень. ЧСС - 110/хв, AT - 100/60, ЧД - 24/хв. Який найбільш вірогідний діагноз? The affected man works at a chemical plant for the production of nitric acid. He complains of lacrimation, runny nose, a sore throat. Within 12 hours, shortness of breath, a feeling of lack of air appeared . Objectively established: the patient's position is semi-sitting, the skin is cyanotic. During auscultation - dry and wet rales over the entire surface of the lungs. Heart rate - 110/min, AT - 100/60, BP - 24/min. What is the most likely diagnosis ?

Гостра серцева недостатність Acute heart failure

Колапс Collapse

Гостре отруєння нігрогазами Acute black gas poisoning

Гостре отруєння оксидом вуглецю Acute carbon monoxide poisoning

Астматичний статус Asthmatic status

143 / 147
Хворий чоловік 43 років скаржиться на різкий біль у задньому проході під час дефекації, наявність слизу з домішками крові в калі, періодичні закрепи. Хворіє протягом 3 років, як після гострої їжі відчув біль та набряк в анальній ділянці. З місяці тому лікувався в стаціонарі через відновлення болю. Зараз має підвищення температури тіла до 37,8°С. Об’єктивно встановлено: збільшені, набряклі гемороїдальні вузли темно-синього кольору. Під час пальцевого обстеження прямої кишки хворий відчуває болючість в анальній ділянці. Поставте найбільш імовірний діагноз: A sick 43-year-old man complains of sharp pain in the anus during defecation, the presence of mucus with blood impurities in the stool, periodic constipation. He has been ill for 3 years, as after after a spicy meal, felt pain and swelling in the anal area. Since a month ago, he was treated in a hospital due to the return of pain. Now he has an increase in body temperature to 37.8°C. Objectively established: enlarged, swollen hemorrhoidal nodes of dark blue color. During digital examination of the rectum, the patient feels pain in the anal region. Make the most likely diagnosis:

Хронічна тріщина прямої кишки Chronic rectal fissure

Хронічний парапроктит Chronic paraproctitis

Гостра тріщина прямої кишки Acute rectal fissure

Тромбоз гемороїдальних вузлів Thrombosis of hemorrhoidal nodes

Гострий парапроктит Acute paraproctitis

144 / 147
Хворий чоловік 74 років, який перебував у відділенні інтенсивної терапії, раптом зблід, втратив свідомість. Зіниці розширились. Пульс на магістральних артеріях відсутній. На екрані кардіомонітора з’явилась крупнохвильова лінія. Які заходи треба вжити насамперед? A 74-year-old man, who was in the intensive care unit, suddenly turned pale, lost consciousness. The pupils were dilated. There was no pulse on the main arteries. The heart monitor screen showed large-wave line. What measures should be taken first?

Забезпечити прохідність дихальних шляхів Ensure patency of airways

Провести дефібриляцію Perform defibrillation

Покликати на допомогу Call for help

Розпочати штучне дихання Start artificial respiration

Розпочати непрямій масаж серця Start indirect heart massage

145 / 147
У хворого чоловіка з проникаючим пораненням грудної клітки крововтрата 40% ОЦК. В анамнезі вказано на численні гемотрансфузії, більшість з яких супроводжувалась гемотрансфузійними реакціями. Виберіть компонент крові для переливання: A sick man with a penetrating chest wound has a blood loss of 40% BCC. The anamnesis indicates numerous blood transfusions, most of which were accompanied by blood transfusion reactions. Select the blood component for transfusion:

Кріопреципітат Cryoprecipitate

Відмиті еритроцити Washed erythrocytes

Еритроцитарна суспензія Erythrocyte suspension

Свіжозаморожсна плазма Fresh-frozen plasma

Концентрат тромбоцитів Platelet concentrate

146 / 147
До медичної роти бригади із вогнища ураження фосфорорганічними отруйними речовинами доставлений постраждалий. В якому виді і об’ємі буде проводитись спеціальна обробка такого ураженого на цьому етапі? A victim was brought to the brigade's medical company from the site of organophosphorus poisoning. In what form and volume will the special treatment of such a victim be carried out at this stage?

Повна санітарна обробка та повна дегазація обмундирування Full sanitation and full degassing of uniform

Часткова санітарна обробка та повна дегазація обмундирування Partial sanitation and full degassing of uniform

Часткова санітарна обробка та часткова дегазація обмундирування Partial sanitation and partial degassing of uniform

Поточна санітарна обробка та заключна дегазація Current sanitation and final degassing

Повна санітарна обробка та часткова дегазація обмундирування Full sanitation and partial degassing of uniform

147 / 147
У новонародженої дитини після забезпечення прохідності дихальних шляхів з’явилося адекватне самостійне дихання, частота дихання - 40/хв., частота серцевих скорочень - 102/хв., акроціаноз. Які Ваші наступні дії? After securing the patency of the respiratory tract, the newborn child developed adequate independent breathing, respiratory rate - 40/min, heart rate - 102/min, acrocyanosis . What are your next actions?

ШВЛ шляхом інтубації трахеї Ventilation by tracheal intubation

Дитина не потребує подальшого спостереження The child does not require follow-up

Непрямий масаж серця Indirect heart massage

Дати кисень через лицеву маску Give oxygen through face mask

ШВЛ 100% киснем Ventilation with 100% oxygen